Anda di halaman 1dari 68

Cure

American
Diabetes
Association

Care

Commitment

ADA-SAP
K Module 1
Basic Principles of Management of Type 2 Diabetes

American Diabetes Association


Self-Assessment Program
Multiple Choice Questions Assessment and Educational (Learning) Critiques Components

2007 American Diabetes Association. All rights reserved.

PEI

Developed and Published by: Professional Evaluation, Inc.


Developing Medical Specialty Board Category 1 CME Programs for Over Three Decades
Professional Evaluation Inc.
4 Midland Avenue, Suite 105, Berwyn, PA 19312-1687 | www.proevalinc.com

American Diabetes Association


Self-Assessment Program (ADA-SAP )
TM

K Module 1
Basic Principles of Management of Type 2 Diabetes

Table of Contents
3

Learning Objectives

Accreditation

Faculty

Disclosure Statement

General Instructions

Multiple Choice Questions (MCQs)

25

Appendix

26

I. Diagnosis of impaired glucose tolerance (IGT) and impaired fasting glucose (IFG)

27

II. Diagnosis of type 2 diabetes

28

III. Summary of recommendations for adults with diabetes

34

Educational Critiques

65

Computer-scored Answer Sheet

66

Program Evaluation Questions

Cure

American
Diabetes
Association

Care

Commitment

Dear Health Care Professional:


The American Diabetes Association (ADA), in cooperation with Professional Evaluation, Inc. (PEI),
has developed the American Diabetes Association Self-Assessment Program (ADA-SAP TM) series.
Module 1 of ADA-SAP TM series covers the Basic Principles of Diagnosis and Management of Type 2
Diabetes in adults. Future modules in the ADA-SAP TM series will cover topics such as, combination
therapy for type 2 diabetes, insulin use, and management of co-morbidities. For additional information,
visit PEIs website, www.proevalinc.com.
The multiple choice questions component of Module 1 of ADA-SAP TM was developed by PEI at the
upper level of difficulty similar in type and format to a board certifying examination. It was designed
to objectively assess, strengthen and reinforce your knowledge and to provide you with an in-depth
learning experience of active engagement. Once you have properly completed Module 1 in its entirety
(see General Instructions on page 5), your knowledge of the approach to diagnosis and management
of type 2 diabetes in adults should be greatly enhanced.
The ADA looks forward to your participation in this important new program of continuing education,
as part of your commitment to lifelong learning.
Sincerely,

Michael H. Davidson M.D., FACC, FACP


Program Chair
Chief Medical Officer, PEI
Clinical Professor of Medicine, Director of Preventive Cardiology
The University of Chicago Pritzker School of Medicine

American Diabetes Association Self-Assessment Program

Target Audience
Primary Care Physicians, General Internists, Family
Physicians, Doctors of Osteopathy, Registered Nurses,
Nurse Practitioners, Pharmacists, Dietitians, and
Certified Diabetes Educators.

Learning Objectives
By completing this CME/CE enduring materials
activity (Module 1), participants will be better able to:
K Discuss the pathogenesis of insulin resistance
and pancreatic -cell dysfunction leading to type 2
diabetes.
K Interpret and utilize ADA Guidelines and
treatment algorithms.
K Implement non-pharmacological and pharmacological treatment strategies for hyperglycemia
in type 2 diabetes.
This enduring materials activity was originally
released on November 15, 2007 and terminated
on November 30, 2009.

Faculty
K Chair
Michael H. Davidson, M.D., FACC, FACP
Clinical Professor
Director of Preventive Cardiology
The University of Chicago
Executive Medical Director
Radiant Research
Chicago, IL

K Faculty
Elliot B. Davidson, M.D., FAAFP
Family Medicine
Akron General Medical Center
Akron, OH
Stanley Schwartz, M.D., FACP
Clinical Associate Professor of Medicine
Director, Diabetes Disease Management
University of Pennsylvania Health System
Philadelphia, PA

K Chief Editor and Publisher


Dante S. LaRocca, Ph.D.
President
Professional Evaluation, Inc.
Ambler, PA

K Reviewers
M. Sue Kirkman, M.D.
Vice President, Clinical Affairs
American Diabetes Association
Alexandria, VA
Belinda P. Childs,
ARNP, MN, CDE, BC-ADM
Clinic and Research Coordinator
Mid-America Diabetes Associates, PA
Wichita, KS
Russell D. White,
M.D., FAFP, FACSM, MACE
Professor of Medicine
Director,
Sports Medicine Fellowship Program
University of Missouri
Kansas City School of Medicine
Truman Medical CenterLakewood
Kansas City, MO

This activity was supported by an unrestricted educational grant from Merck & Co., Inc.
American Diabetes Association Self-Assessment Program

Disclosure Statement
All participating faculty, course directors, and planning committee members are required to disclose to the program audience
any financial relationships related to the subject matter of this program. Disclosure information is reviewed in advance in
order to manage and resolve any possible conflicts of interest. The intent of this disclosure is not to prevent a planner or
presenter from being involved in the activity, but rather to provide participants with information on which they can make
their own judgments.
Content Validation Statement
The American Diabetes Associations accredited continuing education activities are based on evidence that is accepted
within the profession of medicine. Scientific research referred to, reported, or used in support or justification of a patient
care recommendation conforms to the generally accepted standards of experimental design, data collection and analysis.
Unapproved Uses of Drugs/Devices
In accordance with requirements of the FDA, the audience is advised that information presented in this continuing
education activity may contain references to unlabeled or unapproved uses of drugs or devices. Please refer to the FDA
approved package insert for each drug/device for full prescribing/utilization information.
Belinda P. Childs, ARNP, MN, CDE, BC-ADM
Research Support: Amylin, Bayer Healthcare, Biodel, Eli Lilly, Novartis, Roche Diagnostic, sanofi-aventis;
Speakers Bureau: Amylin, Eli Lilly; Advisory Panel: Amylin; Consultant: Roche Diagnostic, sanofi-aventis.
Elliot B. Davidson, M.D., FAAFP
Speakers Bureau: AstraZeneca, Novartis, Reliant Pharmaceuticals, Inc.
Michael H. Davidson, M.D., FACC
Research Support: Abbott Laboratories, AstraZeneca Pharmaceuticals, Daiichi-Sankyo, Inc., Merck/Schering-Plough,
Pfizer Laboratories, Reliant Pharmaceuticals, Inc., Roche Pharmaceuticals, sanofi-aventis, Takeda Pharmaceuticals;
Speakers Bureau: Abbott Laboratories, AstraZeneca Pharmaceuticals, Daiichi-Sankyo Inc., Merck & Co. Inc., Merck
Schering Plough, Pfizer Inc., Reliant Pharmaceuticals, Inc., Takeda Pharmaceuticals; Consultant: Abbott Laboratories,
AstraZeneca Pharmaceuticals, Daiichi-Sankyo Inc., Merck & Co. Inc., Merck Schering-Plough, Pfizer Inc., Reliant
Pharmaceuticals, Inc., Takeda Pharmaceuticals.
M. Sue Kirkman, M.D.
Disclosed no conflict of interest.
Dante S. LaRocca, Ph.D.
Disclosed no conflict of interest.
Stanley Schwartz, M.D., FACP
Speakers Bureau: Amylin, Eli Lilly, Merck & Co. Inc., Novo Nordisk, Inc., Pfizer Inc., sanofi-aventis, Takeda
Pharmaceuticals; Advisory Panel: Eli Lilly, Takeda Pharmaceuticals.
Russell D. White, M.D.
Speaker Bureau: American Academy of Family Physicians, The National Procedures Association, University of MissouriKansas City; Consultant: Novo Nordisk, Inc.

American Diabetes Association Self-Assessment Program

General Instructions for Completion of ADA-SAP


Module 1 of ADA-SAP consists of the following components:
K Multiple Choice Questions (MCQs) Assessment Component
This consists of 75 clinical problem-solving MCQs. Your task is to select the one lettered option among
four or five offered that BEST answers each MCQ.
K Appendix
This is a comprehensive summary of the recommendations of the ADA for diagnosis and management
of type 2 diabetes. It is based on peer-reviewed publications and is designed to assist you to complete
the Assessment and Educational Components of ADA-SAP.
K Answer Sheet
An answer sheet is provided for you to record your answers as you work through the MCQs component of
the program. Refer to this answer sheet to assess your performance while reading the Critiques component.
K Educational (Learning) Critiques Component
IN ORDER TO MAXIMIZE YOUR LEARNING EXPERIENCE IT IS IMPORTANT THAT YOU DO
NOT LOOK AT THE EDUCATIONAL CRITIQUES COMPONENT UNTIL AFTER YOU HAVE
COMPLETED MARKING YOUR ANSWERS FOR THE MCQs ON THE ANSWER SHEET. The
critiques component contains detailed explanations for the correct and incorrect answers for the MCQs based
on the most current peer-reviewed published information. Once you have read the Educational Critiques, the
bibliographic references should be utilized as follow-up study for those MCQs which you answered incorrectly.
The critiques are the teaching and learning component of ADA-SAP. They are to be used in combination
with the MCQ assessment component to provide you with a positive, active learning experience.
DO NOT CHANGE THE ANSWERS MARKED ON YOUR ANSWER SHEET WHILE READING
THE CRITIQUES. THE INTENT OF ADA-SAP IS LEARNING NEW KNOWLEDGE AND
REINFORCING PREVIOUSLY LEARNED KNOWLEDGE.
K Program Feedback
After you have completed the MCQ and Critique components , you may email any comments about the content
or format of this Self-Assessment Program to info@proevalinc.com.

American Diabetes Association Self-Assessment Program

American Diabetes Association


Self-Assessment Program
Multiple Choice Questions (MCQs)

American Diabetes Association Self-Assessment Program

MCQs

Multiple Choice Questions (MCQs)


Items 175
Directions
Items 175 consist of a question or an incomplete statement followed by three, four or five lettered options.
Your task is to select the one BEST lettered option that answers each item. After you have selected the one
lettered option that BEST answers each item, completely blacken the corresponding lettered circle for that
item on Section A of the answer sheet. Please use only a #2 soft lead pencil.

Complex Case Highlight (Items 110)


A 35-year-old African-American man presents to you for his annual physical exam. He has no significant
complaints but is concerned because his older sister was just diagnosed with type 2 diabetes. His father also
had type 2 diabetes and died at age 54 of a myocardial infarction. He has two sons age 10 and 12, both L120%
of ideal weight for their height. He is 60 and weighs 210 lbs (BMI 27 kg/m2); blood pressure is 138/88 mm Hg.
Current lab results for this patient are as follows:
Total Cholesterol

210 mg/dL

Triglycerides

150 mg/dL

HDL-C

40 mg/dL

LDL-C

140 mg/dL

Glucose

96 mg/dL

K Items 16
For each numbered laboratory result (16), (if confirmed by repeat testing), select the one lettered diagnosis for this
patients problem (A, B, C, D) MOST likely associated with it. Each lettered diagnosis may be selected once, more
than once, or not at all.
(A) Impaired fasting glucose (IFG).
(B) Impaired glucose tolerance (IGT).
(C) Diabetes.
(D) None of the above.
1.

Fasting plasma glucose 120 mg/dL.

2.

2-hour plasma glucose 190 mg/dL.

3.

Fasting plasma glucose 130 mg/dL.

MCQs

American Diabetes Association Self-Assessment Program

4.

2-hour plasma glucose 210 mg/dL after a dose of 75 g of anhydrous glucose dissolved in water.

5.

Non-fasting glucose (he ate breakfast 4 hours before) 160 mg/dL.

6.

The same patient but now with a history of polyuria and a non-fasting glucose of 210 mg/dL.

7.

Which one of the following statements regarding this patient is CORRECT?


(A) According to ADA guidelines, since his fasting glucose is l100 mg/dL, testing should be repeated
at 3-year intervals.
(B) According to the NCEP ATP III guidelines, he is at his LDL goal for his risk category.
(C) According to the JNC7, he does not require drug therapy for hypertension.
(D) According to the ADA guidelines, an oral glucose tolerance test (OGTT) is recommended to potentially
diagnose diabetes in patients with a strong family history.
(E) According to ADA guidelines, his two sons should have their fasting plasma glucose measured and
repeated every 2 years.

Upon the recommendation of your patient, his sister comes to see you for treatment of her diabetes. His sister is
39 years old and was diagnosed with diabetes at a health fair in which she had a non-fasting glucose of 180 mg/dL
approximately 6 weeks ago. She has 5 children ages 1020 years old. At birth, all her children weighed at least
9 lbs. She has a history of increased blood pressure for the past 5 years and has tried to watch her salt intake and
lose weight but takes no medication. For the past 6 weeks, she has restricted her caloric intake and started walking
around the block at least 5 times per week. She believes she has lost a few pounds but complains of feeling
sluggish and she urinates more than usual. Her labs are as follows:
Total Cholesterol

180 mg/dL

Triglycerides

200 mg/dL

HDL-C

38 mg/dL

LDL-C

102 mg/dL

Fasting Glucose

170 mg/dL
7.2%

A1C
Creatinine

1.1 mg/dL

BUN
ALT

48 (normal l40)

AST

50 (normal l45)

Height

58

Weight

170 lbs

Blood Pressure

20

138/88 mm Hg

American Diabetes Association Self-Assessment Program

MCQs

8.

According to the ADA guidelines, which one of the following statements regarding the patients sister
is CORRECT?
(A) Before initiating drug therapy, the patient should undergo an exercise stress test to rule out significant
coronary artery disease and, if normal, an aerobic exercise program should be prescribed for at least
60 minutes/day 5 times per week.
(B) Start a low-carbohydrate diet (restricting total carbohydrates to l130 g/day) to induce short-term weight
loss and thereby improve glycemic control.
(C) Start metformin 1000 mg/day in conjunction with a weight reduction meal plan and an exercise program.
(D) Start therapy with a sulfonylurea because she has symptomatic hyperglycemia and this medication will
result in more rapid reduction in her elevated glucose level.
(E) Start with a glitazone because she has a very low HDL (l50 mg/dL) for a woman.

9.

In addition to treatment for hyperglycemia, which one of the following statements is CORRECT regarding
the management of her other cardiovascular risk factors according to AHA recommendations?
(A) Since she has a systolic blood pressure of 130139 mm Hg and a diastolic blood pressure of
8089 mm Hg and has failed lifestyle and behavior therapy, she should be treated with pharmacological
agents that block the rennin-angiotensin system.
(B) Since she is less than 40 years of age, statin therapy is not recommended.
(C) Since she does not have overt CVD, her LDL goal is l130 mg/dL rather than l100 mg/dL.
(D) For a women with diabetes, her triglyceride goal is l100 mg/dL and her HDL goal is L40 mg/dL.
(E) A fibrate is recommended as the drug of first choice to treat her dyslipidemia because she has elevated
triglycerides.

10. For a patient with newly-diagnosed type 2 diabetes, which one of the following ancillary tests is recommended?
(A) Urinary evaluation for microalbuminuria.
(B) A dilated and comprehensive eye examination by an ophthalmologist or optometrist.
(C) Screening for distal symmetric polyneuropathy.
(D) Comprehensive foot exam.
(E) All of the above.

K Items 1112
A 55-year-old African-American female school teacher whose mother and father both developed type 2 diabetes
in their 50s presents to you for an evaluation to determine her risk for developing diabetes mellitus. She has
4 children and during the pregnancy of her fourth child, she developed gestational diabetes. She is a non-smoker
and sedentary. She is 55, weighs 200 lbs, and blood pressure is 128/78 mm Hg. Fasting labs are as follows:
Total Cholesterol

200 mg/dL

Triglycerides

100 mg/dL

HDL-C

50 mg/dL

LDL-C

130 mg/dL

Glucose

110 mg/dL

MCQs

American Diabetes Association Self-Assessment Program

She takes no medications including aspirin. She brings with her a Personal Health Decisions report from
www.diabetes.org/diabetesphd, which is an appraisal on the ADA website for assessing the risk of developing
diabetes, heart disease, or a stroke over the next 30 years.
11. Based on the Personal Health Decisions (Diabetes PHD) appraisal, she has a 38% chance of developing type
2 diabetes. If she loses 10% of her body weight (approximately 20 lbs), her risk of developing type 2 diabetes
would be which one of the following?
(A) Modified to l10%.
(B) Modified to l1120%.
(C) Modified to l2130%.
(D) Her risk remains L35% because she is still significantly overweight.
(E) Due to her history of gestational diabetes, weight loss would be ineffective in modifying her risk
of developing type 2 diabetes.
12. According to the ADA Guidelines for the prevention or delay of type 2 diabetes, which one of the following
next steps would be MOST appropriate?
(A) Initiate therapy with metformin simultaneously with a weight reduction meal plan or exercise program.
(B) Perform a 2-hour oral glucose tolerance test and only initiate metformin if her 2-hour post-prandial
glucose is greater than 180 mg/dL.
(C) Instruct her on a weight reduction meal plan with exercise. If she fails to lose weight and her fasting
glucose remains below 126 mg/dL, medication remains inappropriate.
(D) Instruct her on a weight reduction meal plan program with exercise and consider initiation of metformin
in 612 months if she has not lost significant amounts of weight and her fasting glucose remains greater
than 100 mg/dL.
(E) Instruct the patient on a weight reduction meal plan with exercise with the addition of a weight loss
medication, such as orlistat.

K Item 13
A 40-year-old Hispanic woman with newly-diagnosed type 2 diabetes mellitus has been on a weight reduction
meal plan for the past 3 months and has lost 5 lbs. She is on simvastatin 20 mg, enalapril 10 mg, and aspirin 81 mg.
Her labs are as follows:
Total Cholesterol

160 mg/dL

Triglycerides

200 mg/dL

HDL-C

40 mg/dL

LDL-C

70 mg/dL

Glucose

180 mg/dL

A1C

9.0%

Height

51

Weight

150 lbs

Waist Circumference
Blood Pressure
10

37
128/80 mm Hg

American Diabetes Association Self-Assessment Program

MCQs

13. According to the American Diabetes Association (ADA) treatment algorithm for type 2 diabetes,
the MOST appropriate treatment option for this patient is which one of the following?
(A) Since she has lost 5 lbs, oral hypoglycemic agents should be withheld until weight loss treatment
has been maximized.
(B) Initiate metformin therapy as the first-line treatment for type 2 diabetes.
(C) Since she has significant glucose elevation, a sulfonylurea should be initiated because these oral
hypoglycemic agents are the most potent in treating hyperglycemia.
(D) Because she has elevated triglyceride and low HDL levels, a thiazolidinedione (TZD) should be initiated.
(E) She should receive treatment with sitagliptin because it has a low side-effect profile and is a category B
drug for women of child-bearing potential.

K Items 1418
For each numbered clinical scenario or pharmacologic effect (1418), select the one MOST appropriate lettered
PPAR agonist treatment option associated (A, B, C, D, E). Each lettered PPAR agonist treatment option may be
selected once, more than once, or not at all.
(A) Fenofibrate.
(B) Gemfibrozil.
(C) Thiazolidinedione.
(D) None of the above.
(E) All of the above.
14. Contraindicated in a man with cardiovascular disease and NYHA Class I heart failure.
15. Inhibits statin glucuronidation leading to elevated statin levels.
16. Dosage adjustment is necessary in patients with moderate to severe renal impairment.
17. Not contraindicated in a woman with diabetes and fatty liver disease in whom the ALT is 75 U/L
(normal l40) and the AST 46 U/L (normal l36).
18. Causes a fall in the hematocrit.

K Items 1923
For each numbered physiological or pathophysiological effect (1923), select the one MOST accurate lettered
adipocytokine (A, B, C, D, E) associated with that effect. Each lettered adipocytokine may be selected once,
more than once, or not at all.
(A) Adiponectin.
(B) TNF- (tumor necrosis factor-alpha).
(C) Free fatty acids.
(D) Both TNF- and free fatty acids.
(E) PAI-1 (plasminogen activator inhibitor-1).
MCQs

American Diabetes Association Self-Assessment Program

11

19. Increases insulin sensitivity.


20. Inhibits monocyte adhesion to vascular wall endothelium.
21. Causes hyperinsulinemia by interfering with hepatic insulin degradation.
22. Contributes to insulin resistance.
23. Localizes in the plaque where it is thought to promote coagulation.
24. Which one of the following statements is CORRECT regarding the therapy of patients with type 2 diabetes
mellitus with biguanides?
(A) They will be the drug of choice in patients with compromised left ventricular function due to their low
risk of hypoglycemia.
(B) They should not be used in patients with significant renal impairment.
(C) They should not be used in combination with insulin due to the increased risk for side-effects.
(D) They lose efficacy when combined with a thiazolidinedione.
(E) They are best tolerated and most effective when taken on an empty stomach.
25. Which one of the following mechanisms MOST likely contributes to accelerated vascular disease associated
with insulin resistance?
(A) The direct effect of elevated insulin levels on the vessel wall.
(B) A pro-inflammatory state.
(C) Accelerated fatty acid oxidation in vascular smooth muscle cells.
(D) Reduced plasminogen activator inhibitor-1 (PAI-1) levels.
(E) Reduced homocysteine levels.
26. Which one of the following statements is CORRECT regarding home monitoring of blood glucose?
(A) It is only useful for patients on insulin.
(B) In the most aggressively managed patients it should be done every 6 hours.
(C) It is generally not necessary in patients with type 2 diabetes mellitus.
(D) Measuring a fasting blood glucose is most helpful for adjusting a basal insulin dose.
(E) None of the above.

12

American Diabetes Association Self-Assessment Program

MCQs

K Items 2731
For each numbered oral hypoglycemic agent (2731), select the one lettered mechanism of action (A, B, C, D, E)
which is MOST important for its efficacy in lowering blood glucose. Each lettered mechanism of action may be
selected only once.
(A) Increases insulin secretion.
(B) Delays glucose absorption in the intestine.
(C) Suppresses glucose production in the liver.
(D) Increases glucose uptake into muscle.
(E) Inhibits the enzyme that degrades GLP-1 (glucagon-like protein-1).
27. Sulfonylureas.
28. Thiazolidinediones.
29. Biguanides.
30. Alpha-glucosidase inhibitors.
31. Sitagliptin.

K Items 3235
For each numbered undesirable side-effect (3235), select the one lettered class of oral hypoglycemic agent
(A, B, C, D, E) MOST closely associated with it. Each lettered class of oral hypoglycemic agent may be
selected once, more than once, or not at all.
(A) Sulfonylureas.
(B) Thiazolidinediones.
(C) Biguanides.
(D) Alpha-glucosidase inhibitors.
(E) None of the above.
32. Increased fluid retention and edema.
33. Nausea; lactic acidosis.
34. Hypoglycemia.
35. Gas, bloating.

MCQs

American Diabetes Association Self-Assessment Program

13

36. A female patient with type 2 diabetes mellitus, weighing 300 lbs, asks you about treatment options
but is afraid of using medication for the treatment of diabetes. Which one of the following is the MOST
effective method of achieving significant weight loss in patients with diabetes mellitus without using
antihyperglycemia medications?
(A) Diets emphasizing fewer calories.
(B) Increased physical activity.
(C) Diets emphasizing a reduction in saturated fats.
(D) Bariatric surgery.
(E) Medications for weight loss.
37. An overweight man with type 2 diabetes mellitus and peripheral neuropathy asks you about starting an
exercise program to lose weight. Which one of the following is NOT a beneficial effect of incorporating
increased physical activity and weight loss into the regimen of this patient?
(A) Reduction in blood glucose.
(B) Reduction in blood pressure.
(C) Improvement in lipid profile.
(D) Improved sense of control over the illness.
(E) Will achieve less injuries related to neuropathy.
38. Metformin is often selected when starting a patient with newly-diagnosed diabetes on medication for the first
time. Which one of the following is NOT an advantage with metformin over other medications?
(A) It rarely causes hypoglycemia.
(B) It rarely induces weight gain.
(C) It is associated with no serious side-effects.
(D) It is generally inexpensive.
(E) When dosed effectively it will usually lower A1C more than 1%.
39. Sulfonylureas are frequently prescribed as part of the regimen for patients with type 2 diabetes.
Which one of the following concerns about sulfonylureas is INCORRECT?
(A) Have a tendency to cause weight gain.
(B) May cause hypoglycemia, especially the longer-acting agents.
(C) Have been consistently shown to increase mortality from cardiovascular disease.
(D) May interact with other sulfa drugs or glinides to increase risk of hypoglycemia.
(E) May cause a rash.

14

American Diabetes Association Self-Assessment Program

MCQs

40. The glinides (e.g., repaglinide and nateglinide) are a recommended substitute for sulfonylureas for use
as monotherapy in patients with type 2 diabetes mellitus with which one of the following?
(A) Congestive heart failure.
(B) Liver disease.
(C) Renal insufficiency.
(D) Associated hypertriglyceridemia.

K Items 4145
For each numbered anti-hyperglycemic agent (4145), select one lettered primary mechanism of action
(A, B, C, D, E) associated with it. Each lettered primary mechanism of action may be selected only once.
(A) Reduces gluconeogenesis in the liver.
(B) Stimulates post-prandial insulin secretion.
(C) Delays the absorption of carbohydrates in the gut.
(D) Enhances insulin sensitivity in the peripheral tissues.
(E) Inhibits the breakdown of incretins.
41. Metformin.
42. Glitazones.
43. Alpha-glucosidase inhibitors.
44. Glinides.
45. Sitagliptin.

K Items 4648
A 50-year-old woman with type 2 diabetes mellitus is seen in the office with a blood pressure of 150/90 mm Hg
and the lipid profile shown below. She has a normal amount of albumin in her urine and normal renal function.
Total Cholesterol

215 mg/dL

Triglycerides

120 mg/dL

HDL-C

51 mg/dL

LDL-C

140 mg/dL

MCQs

American Diabetes Association Self-Assessment Program

15

46. If her doctor elects to start her on medication for her blood pressure, which one of the following classes
of antihypertensive agents would be BEST to reduce her chance of progression to microalbuminuria?
(A) Alpha blockers.
(B) Diuretics.
(C) ACE inhibitors.
(D) Beta blockers.
(E) Calcium channel blockers.
47. She is reluctant to take a statin. Which one of the following statements is INCORRECT regarding statins
and the prevention of CVD?
(A) Statins have been shown to reduce the chances of a CV event in individuals that have no known
CVD (primary prevention).
(B) All the large published studies which evaluated statins ability to prevent all cause mortality have shown
a benefit.
(C) Statins have been shown to reduce the chances of a second CV event in individuals that have known
CVD (secondary prevention).
(D) Statins have been shown to reduce the chances of having a CV event whether the LDL level is
L116 mg/dL or l116 mg/dL
(E) The American Diabetes Association recommends all patients with type 2 diabetes mellitus reduce
LDL levels to 100 mg/dL or lower.
48. An alpha-glucosidase inhibitor, such as acarbose would be LEAST appropriate for the treatment of a patient
with type 2 diabetes mellitus and which one of the following clinical characteristics?
(A) A1C level of 7.5 mg/dL with kidney disease.
(B) A1C level of 7.5 mg/dL with liver disease.
(C) A1C level of 8.0 mg/dL with a tendency toward hypoglycemia.
(D) A1C level of 12.0 mg/dL with low tolerance for side-effects.
(E) A1C level of 7.2 mg/dL with concerns about systemic side-effects.
49. Insulin should be considered, along with lifestyle modifications, as the initial therapy in a newly-diagnosed
patient with type 2 diabetes mellitus in all of the following situations EXCEPT:
(A) Fasting blood glucose level L250 mg/dL.
(B) A1C L10%.
(C) BMI l27 kg/m2.
(D) Presence of ketonuria.
(E) Symptomatic hyperglycemia (polyuria, polydypsia).

16

American Diabetes Association Self-Assessment Program

MCQs

50. A 43-year-old overweight hypertensive woman is newly-diagnosed with type 2 diabetes mellitus, with a A1C
of 8.5%. Which one of the following is the BEST choice for initial therapy in addition to weight reduction?
(A) Metformin.
(B) A sulfonylurea.
(C) A glitazone.
(D) Inhaled insulin.
(E) Sitagliptin.
51. When initiating insulin therapy in a patient with type 2 diabetes, which one of the following approaches
is recommended?
(A) Start with a bedtime dose of an intermediate-acting form or a once daily, long-acting form.
(B) Start with a 2-shot regimen of a long-acting form.
(C) Start with a 3-shot regimen of a short-acting form.
(D) Start with a higher dose and titrate down.
(E) Make dosing changes infrequently to give time for the patient to adjust to the dose.
52. A 48-year-old man with type 2 diabetes, treated with a basal dose of long-acting insulin and oral medications,
is struggling to control his post-prandial blood glucose. When adding rapid-acting insulin with meals to a
basal insulin dosage, it is important to remember which one of the following additional adjustments?
(A) Reduce the basal insulin dose to avoid hypoglycemia.
(B) Discontinue any insulin secretogogue, like a sulfonylurea or glinide.
(C) Increase the dose of the insulin secretogogues to enhance the effect of the injected insulin.
(D) Be sure to add an ACE inhibitor to improve renal function.
(E) Reduce the total carbohydrate intake by 30% to avoid hyperglycemia.

K Items 5354
A 51-year-old woman with type 2 diabetes has been treated for 3 months with lifestyle modifications
and metformin at 1000 mg b.i.d., but her A1C level is still L7%.
53. Which one of the following steps would be LEAST appropriate for her at this time?
(A) Add a glitazone (rosiglitazone or pioglitazone).
(B) Add glipizide.
(C) Add a single dose of basal insulin.
(D) Increase the metformin to 1000 mg t.i.d.
(E) Add glyburide.

MCQs

American Diabetes Association Self-Assessment Program

17

54. The patient is reluctant to intensify her therapy because she feels fine. In discussing the benefits of reducing
her A1C level, you consider how much benefit she can derive from additional reduction. According to
the United Kingdom Prospective Diabetes Study (UKPDS), for every 1% reduction in A1C level in the
average patient with type 2 diabetes, which one of the following results would NOT be expected?
(A) A 21% decrease in any diabetes-related endpoint.
(B) A 14% decrease in the risk of myocardial infarction.
(C) A 12% decrease in the risk of stroke.
(D) A 37 % decrease in the risk of microvascular disease.
(E) A 42% decrease in the risk of heart failure.
55. Which one of the following vaccines is recommended for all patients with diabetes between the ages
of 1864 years?
(A) Meningococcal.
(B) Pneumococcal once and yearly influenza.
(C) Hepatitis A.
(D) Hepatitis B.
(E) Pneumococcal every 5 years and influenza yearly.
56. On average, patients with type 2 diabetes mellitus have lost approximately what percentage of their pancreatic
beta-cell (-cell) function by the time their diabetes is diagnosed?
(A) 20%.
(B) 35%.
(C) 50%.
(D) 70%.
(E) 90%.
57. Which one of the following is NOT commonly associated with hypoglycemia?
(A) Anxiety.
(B) Tachycardia.
(C) Shakiness.
(D) Death.
(E) Diaphoresis.

18

American Diabetes Association Self-Assessment Program

MCQs

K Items 5859
The pathophysiology of type 2 diabetes mellitus requires both insulin resistance and pancreatic -cell dysfunction.
The major targets for insulin are the liver, skeletal muscle, and adipose tissue.
58. Which one of the following statements is INCORRECT regarding insulin resistance in patients with
type 2 diabetes mellitus?
(A) The normal suppression of gluconeogenesis by insulin in the liver is impaired.
(B) The normal uptake of glucose by skeletal muscle is impaired.
(C) The normal glucose disposal, which includes glucose oxidation and glycogen synthesis, by skeletal
muscle, is impaired.
(D) Excess free fatty acids (FFAs) impair -cell function.
(E) The normal degradation of triglycerides into FFAs by adipose tissue is impaired.
59. Which one of the following is NOT a major factor in pancreatic -cell dysfunction in patients with
type 2 diabetes mellitus?
(A) Environmental factors.
(B) Genetic factors.
(C) Lipotoxicity.
(D) Glucotoxicity.
(E) Insulin toxicity.
60. Which one of the following conditions depicted pictorially is NOT associated with type 2 diabetes mellitus?

(A)

(B)

(C)

(D)

MCQs

American Diabetes Association Self-Assessment Program

19

Management of a patient with type 2 diabetes (Items 6171)


A 28-year-old African-American man presented to his primary care physicians office with complaints of knee pain.
In college, he played guard on the football team. He had anterior cruciate ligament surgery after college, stopped
regular exercise, and gradually increased weight.
Past History: no other surgery.
Family History: positive for type 2 diabetes mellitus, hypertension.
Social History: negative for smoking, no drug abuse, occasional alcohol intake.
Review of symptoms: non-contributory.
Triglycerides

152 mg/dL

HDL-C

38 mg/dL

LDL-C

110 mg/dL

Post-prandial Glucoses (PPG)

144 mg/dL & 154 mg/dL

A1C
Blood Pressure

5.6%
130/82 mm Hg

Heart Rate
Respirations
BMI

76 BPM
14
34 kg/m2

Fundi

normal

Pulmonary, cardiac,
abdominal & neurological exams

normal

61. The patient asks how to prevent overt diabetes. Which one of the following therapies has NOT been proven
to prevent diabetes in patients with impaired glucose tolerance?
(A) Lifestyle changes.
(B) Metformin.
(C) Acarbose.
(D) Rosiglitazone.
(E) Ramipril.
62. Diet should never be forgotten as part of the care of patients with diabetes. The optimal mix of macronutrients
is not clear; in fact, a diet consisting of foods which the patient usually eats, modified to fit basic principles,
will ensure compliance. In a patient, such as this 28-year-old African-American man with impaired glucose
tolerance or type 2 diabetes, which one of the following diets is NOT recommended?
(A) A diet that includes sugar alcohols and non-nutritive sweeteners in FDA-approved quantities.
(B) A diet that includes fruits, vegetables, fiber, whole grain, legumes and low fat meals.
(C) A diet that includes fiber.
(D) A diet that includes l20 g of carbohydrate foods daily.
(E) A diet that includes controlled quantities of sucrose-containing foods.
20

American Diabetes Association Self-Assessment Program

MCQs

63. Which one of the following is a characteristic seen in some African-Americans with new-onset type 2 diabetes?
(A) Ability to stop insulin therapy soon after presentation with diabetic ketoacidosis (DKA).
(B) A particular responsiveness to a thiazolidinedione (TZD).
(C) Commonly have anti-islet cell antibodies.
(D) Low incidence of family history of diabetes.
(E) Associated with sickle cell trait.
64. A generally acceptable ideal goal of treatment for this patient might include each of the following EXCEPT:
(A) A1C l7%.
(B) A1C l6% without undue hypoglycemia.
(C) Fasting blood glucose l140 mg/dL.
(D) Peak post-prandial blood glucose l180 mg/dL.
65. This patients A1C level on lifestyle therapy alone was maintained at ~5.8% for about a year, but gradually
rose to p6.5%, and on return to the practice was 7.2%. It was elected to prescribe metformin. According to
ADA guidelines, metformin should be started at the time of diagnosis because of which one of the following?
(A) Effectiveness in decreasing A1C.
(B) Low cost.
(C) Acceptable low risk of side-effects.
(D) Proven outcome benefits.
(E) All of the above.
66. Which one of the following is NOT an effect of therapy with metformin?
(A) It reduces hepatic glucose output.
(B) It reduces hepatic gluconeogenesis.
(C) It reduces insulin levels.
(D) It reduces fasting blood glucose levels.
(E) It increases C-peptide levels.
67. The patient did well for about 2 years on metformin. However the AIC again increased to 7.2% and
pioglitazone was added to metformin. His A1C decreased to 6.4%. However, his A1C increased again
and it was elected to add a third agent when it reached 7.1%. Which one of the following is NOT a logical
choice for a third drug added to this patients treatment?
(A) Glipizide.
(B) Rosiglitazone.
(C) Glargine.
(D) Exenatide.
(E) Sitagliptin.

MCQs

American Diabetes Association Self-Assessment Program

21

68. Which one of the following is NOT an abnormality resulting from increased visceral fat in patients
with the metabolic syndrome?
(A) Increased myocyte insulin resistance.
(B) Increased liver insulin resistance.
(C) Increased adiponectin level.
(D) Increased likelihood for the onset of type 2 diabetes.
(E) Decreased -cell function.
69. Which one of the following statements is INCORRECT regarding post-prandial glycemia?
(A) It is the earliest detectable glycemic abnormality in the majority of patients.
(B) Convincing data exists that treating it will reduce cardiovascular outcomes or other complications
of diabetes.
(C) It contributes to the A1C level of 6.5% to a greater extent than does fasting blood glucose.
(D) It is associated with an increased risk of microvascular complications.
(E) It is associated with an increased risk of macrovascular complications.
70. Primary data supporting the use of analog insulin does NOT include which one of the following?
(A) It mimics normal physiology.
(B) It decreases overnight hypoglycemia.
(C) It reduces A1C level more than does human insulin.
(D) Patient lifestyle will control the disease therapy rather than the disease therapy controlling the lifestyle.
(E) It decreases late post-prandial hypoglycemia.
71. Which one of the following is NOT a likely mechanism for complications in diabetes mellitus?
(A) Increased pentose-phosphate shunt flux.
(B) Increased flux in aldose reductase pathway.
(C) Increased advanced glycosylation end product production.
(D) Increased hexosamine pathway flux.
(E) Increased oxidative stress in mitochondria.
72. Which one of the following statements about treatments of hypertension in patients with type 2 diabetes
is INCORRECT?
(A) Aim for blood pressure under 130/80 mm Hg.
(B) Include an ACE inhibitor or an ARB if possible.
(C) Often requires combination therapy.
(D) Must absolutely contraindicate hydrochlorothiazide (HCTZ) because it may increase blood glucose.
(E) Beta blockade may increase the risk of hypoglycemic unawareness.

22

American Diabetes Association Self-Assessment Program

MCQs

K Items 7375
A logical choice of combination therapy with metformin should depend on individual characteristics of patients
with type 2 diabetes. For each numbered patient being treated with metformin (7375), select the one lettered
additional agent in combination therapy (A, B, C) most appropriate for that patient. Each lettered additional agent
in combination therapy may be selected only once.
(A) Insulin.
(B) Exenatide.
(C) Sitagliptin.
73. A patient with a BMI L35 kg/m2.
74. A patient with A1C L9.6%.
75. A patient who is a long-distance truck driver and fears injections.

MCQs

American Diabetes Association Self-Assessment Program

23

K Notes

24

American Diabetes Association Self-Assessment Program

Appendix
I. American Diabetes Association Guidelines
and Treatment Algorithms At-a-Glance

Appendix

American Diabetes Association Guidelines and Treatment Algorithms At-A-Glance

25

Definition and Description of Diabetes Mellitus


Diabetes mellitus is a group of metabolic diseases characterized by hyperglycemia resulting from defects in insulin
secretion, insulin action, or both. The chronic hyperglycemia of diabetes is associated with long-term damage,
dysfunction, and failure of various organs, especially the eyes, kidneys, nerves, heart, and blood vessels.

I. Diagnosis of impaired glucose tolerance (IGT) and impaired fasting glucose (IFG)
This group is defined as having fasting plasma glucose (FPG) levels M100 mg/dL (5.6 mmol/L) but
l126 mg/dL (7.0 mmol/L) or 2-h values in the oral glucose tolerance test (OGTT) of M140 mg/dL (7.8 mmol/L)
but l200 mg/dL (11.1 mmol/L). Thus, the categories of FPG values are as follows:
K FPG l100 mg/dL (5.6 mmol/L) normal fasting glucose;
K FPG 100125 mg/dL (5.66.9 mmol/L) = IFG;
K FPG M126 mg/dL (7.0 mmol/L) provisional diagnosis of diabetes (the diagnosis must be confirmed,
as described below).
The corresponding categories when the OGTT is used are the following:
K 2-h post load glucose l140 mg/dL (7.8 mmol/L) normal glucose tolerance;
K 2-h post load glucose 140199 mg/dL (7.811.1 mmol/L) = IGT;
K 2-h post load glucose M200 mg/dL (11.1 mmol/L) provisional diagnosis of diabetes (the diagnosis must
be confirmed, as described below).
Patients with IFG and/or IGT are now referred to as having pre-diabetes indicating the relatively high risk
for development of diabetes in these patients.
Table 1. Treatment recommendation for individuals with IFG, IGT, or both.
Population

Treatment

IFG or IGT.

Lifestyle modification (i.e., 510% weight loss and moderate


intensity physical activity p30 min/day).

Individuals with IFG and IGT and any of the following:


K L60 years of age.
K BMI M35 kg/m2.
K Family history of diabetes in first-degree relatives.
K Elevated triglycerides.
K Reduced HDL-C.
K Hypertension.
K A1C L6.0%.

Lifestyle modification (as above) and/or metformin*.

* Metformin 850 mg twice/day.

26

American Diabetes Association Guidelines and Treatment Algorithms At-A-Glance

Appendix

II. Diagnosis of type 2 diabetes


Table 2. Criteria for testing for diabetes in asymptomatic adult individuals.
1. Testing for diabetes should be considered in all individuals at age 45 years and above, particularly in those with a
BMI M25 kg/m2*, and, if normal, should be repeated at 3-year intervals.
2. Testing should be considered at a younger age or be carried out more frequently in individuals who are overweight
(BMI M25 kg/m2*) and have additional risk factors:
K Are habitually physically inactive.
K Have a first-degree relative with diabetes.
K Are members of a high-risk ethnic population (e.g., African-American, Latino, Native American, Asian American,
Pacific Islander).
K Have delivered a baby weighing L9 lbs or have been diagnosed with GDM.
K Are hypertensive (M140/90 mm Hg).
K Have an HDL-C level l35 mg/dL (0.90 mmol/L) and/or a triglyceride level L250 mg/dL (2.82 mmol/L).
K Have PCOS (polycystic ovary syndrome).
K On previous testing, had IGT or IFG.
K Have other clinical conditions associated with insulin resistance (e.g., PCOS or acanthosis nigricans).
K Have a history of vascular disease.
* May not be correct for all ethnic groups.

Table 3. Testing for type 2 diabetes in children.


Criteria
K Overweight (BMI L85th percentile for age and sex, weight for height L85th percentile, or weight L120%
of ideal for height).
Plus any two of the following risk factors:
K Family history of type 2 diabetes in first- or second- degree relative.
K Race/ethnicity (Native American, African-American, Latino, Asian American, Pacific Islander).
K Signs of insulin resistance or conditions associated with insulin resistance (acanthosis nigricans, hypertension,
dyslipidemia, or PCOS).
K Maternal history of diabetes or GDM.
Age of initiation: age 10 years or at onset of puberty, if puberty occurs at a younger age.
Frequency: every 2 years.
Test: FPG preferred.
Clinical judgment should be used to test for diabetes in high-risk patients who do not meet these criteria.

Table 4. Criteria for the diagnosis of diabetes mellitus.


1. Symptoms of diabetes plus casual plasma glucose concentration M200 mg/dL (11.1 mmol/L).
1). Casual is defined as any time of day without regard to time since last meal. The classic symptoms of diabetes include
polyuria, polydipsia, and unexplained weight loss.
OR
2. FPG M126 mg/dL (7.0 mmol/L). Fasting is defined as no caloric intake for at least 8 hours.
OR
3. 2-h post load glucose M200 mg/dL (11.1 mmol/L) during an OGTT. The test should be performed as described by the
World Health Organization, using a glucose load containing the equivalent of 75 g anhydrous glucose dissolved in water.
In the absence of unequivocal hyperglycemia, these criteria should be confirmed by repeat testing on a different day. The third measure (OGTT) is not
recommended for routine clinical use.
Appendix

American Diabetes Association Guidelines and Treatment Algorithms At-A-Glance

27

III. Summary of recommendations for adults with diabetes.


Glycemic Control
A1C

l7.0%*

Preprandial capillary plasma glucose

90130 mg/dL (5.07.2 mmol/L)

Peak post-prandial capillary plasma glucose

l180 mg/dL (l10.0 mmol/L)

Blood pressure

l130/80 mm Hg

Lipids
LDL-C

l100 mg/dL (l2.6 mmol/L)

Triglycerides

l150 mg/dL (l1.7 mmol/L)

HDL-C

L40 mg/dL (L1.0 mmol/L)

Key concepts in setting glycemic goals:


K A1C is the primary target for glycemic control.
K Goals should be individualized.
K Certain populations (children, pregnant women, and elderly) require special considerations.
K More stringent glycemic goals (i.e., a normal A1C, l6%) may further reduce complications at the cost
of increased risk of hypoglycemia.
K Less intensive glycemic goals may be indicated in patients with severe or frequent hypoglycemia.
K Post-prandial glucose may be targeted if A1C goals are not met despite reaching preprandial glucose goals.
* Referenced to a non-diabetic range of 4.06.0% using a DCCT-based assay.
Post-prandial glucose measurements should be made 12 hours after the beginning of the meal, generally peak levels in patients with diabetes.
Current NCEP/ATP III guidelines suggest that in patients with triglycerides M200 mg/dL, the non-HDL-C (total cholesterol minus HDL) be utilized.
The goal is m130 mg/dL (121).
For women, it has been suggested that the HDL-C goal be increased by 10 mg/dL.

Table 5. Detailed diabetes history and evaluation.


Past medical history should include:
K
K
K
K
K
K
K
K
K

Prior glucose values including A1C, symptoms, treatments, and history of self-monitoring.
History of complications: hypoglycemia and/or ketoacidosis including frequency, severity, and causes.
Self-management, self-monitoring education, attitudes, health beliefs.
Mood disorder.
Prior or current infections, particularly skin, foot, dental, and genitourinary.
Current microvascular symptoms and complications (eye, kidney, nerves).
Current macrovascular symptoms and complications (evaluation of cardiovascular risk factors and cardiovascular disease).
Sexual dysfunction.
Review of past/present lifestyle modification
k Eating patterns
k Smoking history
k Diet history
k Alcohol history
k Weight history
k Controlled substances
k Exercise history
K Other medications evaluate for effect on blood glucose.
K Family history of diabetes and CVD.
Referrals
K Eye exam.
K Family planning for women of reproductive age.
K Dietitian for medical nutrition therapy (MNT).

K Diabetes educator.
K Foot specialist.
K Dentist.

Adapted from American Diabetes Association Clinical Practice Recommendations, 2006. Diabetes Care 2006;29(suppl. 1):S9.
28

American Diabetes Association Guidelines and Treatment Algorithms At-A-Glance

Appendix

Table 6. Follow-up testing recommendations with diabetes.


Potential Complications

Test

Frequency of Evaluation

Hyperglycemia

A1C

Measure A1C 2 times/year if at goal,


quarterly if therapy changed, or not at goal

Fasting blood glucose

Varies depending on prescription regime

Self-monitoring of blood glucose

Frequency/timing should be individualized


to facilitate reaching goals

Hypertension

Blood pressure

Measure at every visit

Autonomic Neuropathy
(Orthostatic Hypotension)

Sitting and standing blood pressure

Appropriate patients/each visit

Dyslipidemia

Lipid panel

Test annually More often if not


at goal or when changing therapy

Foot Care

Quantitative somato-sensory threshold


test using Semmes-Weinstein 5.07 (10 g)
monofilament, tuning fork, palpation, and
visual inspection

Perform a visual inspection at each


routine visit for people with neuropathy.

Nephropathy

Annual comprehensive foot exam

Serum creatinine

Annually

Albumin/Creatinine ratio

Type 2 diabetes mellitus Annual screening

(Random spot urine)

Type 1 diabetes mellitus Annual screening


with duration of diabetes M5 years
Type 1 & 2 diabetes mellitus 2 of 3
specimens within a 36 month period
should be abnormal before considering
a new diagnosis

Retinopathy

Comprehensive eye exam

Type 2 diabetes mellitus Annual dilated &


comprehensive exam
Type 1 diabetes mellitus Initial dilated &
comprehensive exam within 5 years of onset;
annual exams thereafter

Periodontal disease (common


in type 2 diabetes mellitus)

Dental evaluation

Annually

Influenza and pneumonia

Immunization

Annual influenza vaccination

Adapted from American Diabetes Association Clinical Practice Recommendations, 2006. Diabetes Care 2006;29(suppl. 1):S442.

Appendix

American Diabetes Association Guidelines and Treatment Algorithms At-A-Glance

29

Diagnosis
Lifestyle Intervention + Metformin
No
Add Basal Insulin
(Most effective)
A1C M7%

No

Intensify Insulin
No

A1C M7%

A1C M7%

Yes*

Add Sulfonylurea
(Least expensive)
No

Yes*

Add Glitazone
Yes*

A1C M7%

Add Glitazone
(No hypoglycemia)
A1C M7%

No

Yes*

Add Basal Insulin


No

Yes*

Add Sulfonylurea

A1C M7%

Yes*

Add Basal or Intensify Insulin


Intensive Insulin + Metformin +/- Glitazone
Figure 1. Algorithm for the metabolic management of type 2 diabetes. Reinforce lifestyle intervention at every visit.
* Check A1C every 3 months until l7% and then at least every 6 months.
Although three oral agents can be used, initiation and intensification of insulin therapy is preferred based on effectiveness and expense.

Each anti-diabetic medication class possesses differences with regard to contraindications, adverse effects and
mechanisms of action that may dictate which medication is the best for an individual patient (Table 7). Sulfonylurea therapy may be most useful in thin patients with insulinopenia, while metformin may be most useful in
obese patients with dyslipidemia. The -glucosidase inhibitors and meglitinides may be most useful in patients
with exaggerated post-prandial increases in blood glucose, however the -glucosidase inhibitors can be used with
sulfonylureas while the meglitinides can be used with metformin. The thiazolidinediones may be most useful in
patients with insulin resistance or azotemia. Insulin may be useful in patients who will not be able to reach their
glycemic goal with the use of the other adjunct therapies just mentioned.
Exenatide injection is the first in a new class of drugs for the treatment of type 2 diabetes called incretin mimetics.
Exenatide increases glucose-dependent insulin secretion, suppresses inappropriately elevated glucagon secretion,
and slows gastric emptying. Exenatide is indicated as adjunctive therapy to improve glycemic control in patients
with type 2 diabetes who are taking sulfonylurea, metformin, a TZD or combinations of these agents but have not
achieved adequate glycemic control.

30

American Diabetes Association Guidelines and Treatment Algorithms At-A-Glance

Appendix

Table 7. Drug choices based on drug & patient characteristics

Metformin

Thiazolidinedione Sulfonylurea

Dosing
frequency

23x daily

Pioglitazone:
1x daily

Speed of action
FBS/ppg

Weeks/weeks

Sitagliptin
(Dpp-4
inhibtor)

Acarbose/
Miglitol

Glime- Glipizide/ * 1 with


pramide: Glyburide: each meal
1x daily 2x daily

Inject before
breakfast
and dinner

1x daily

3x daily
with meals

Weeks

Day

Day

Day

Day

Months

xx
xxx
xxx

xx
xxx
xxx

xx
xxx
xxx

xxx
xx
xxx

xx

xx
x
xx

xxx
x
x

Concern/
side-effects

GI upset, lactic
acidosis: avoid in
patients with CHF,
liver disease, acute
illness, other
acidosis, Cr >1.4,
hospitalized
patients and hold
2 days after
IV contrast until
Creatinine stable

Edema
wt gain +++
Avoid with no salt
diet & avoid NSAID
Channel blocker,
COX 2 inhibitors
CHF 1%

wt gain ++/+

Nausea
None
Avoid by stop
eating when
feel full

GI sideeffects
Treat hypo
with glucose

Hypoglycemic
risk when
used alone

None

None

+++

+ +/ +

None

None

Monitor for

Creatinine &
Edema/
creatinine clearance Heart Failure

Hypoglycemia

Hypoglycemia Hypoglcemia Hypglocemia LFTs


if use with
if use with
SU/insulin
SU/insulin

Weight

OR NC

OR NC

Lipids
HDL
LDL
TG

NC

Insulin levels

(less with glim.)

Elderly

AVOID if >70 years


(because
creatinine
clearance) avoid
Cr decrease <70

No special risks

Preg risk in pts.


with Polycystic
Ovary Syndrome

Efficacy
Post-prandial
FBS
HgA1

Rosiglitazone:
2x daily

Repaglinide/
Nateglinide* Exenatide

xxx
x
x

xx

Neutral

NC

Decrease

Decrease

NC
NC
NC

Glucosedependent
rise

Glucosedependent
rise

NC

Watch for
Hypoglycemia

Watch for
Hypoglycemia

Ideal

Ideal

No special
risks (GI sideeffects may
be beneficial)

both
NC

NC

Decrease OR NC

Decrease

None

Other Drug Information


Primary
failure rate

20%

20%

20%

20%

Cost

xx

xxx

Xgeneric, XXbrand

xx

? long term
complications

AGE products
in vitro
endothelial
dysfunction

endothelial
dysfunction
Preserve beta
cell function.
Only for Pio=
advantageous
for lipid profile

Appendix

No data
xxx

xxx

xx

American Diabetes Association Guidelines and Treatment Algorithms At-A-Glance

31

Bibliography
1. Byetta Prescribing Information. February 2007. www.byetta.com. Accessed March 2007.
2. Fineman MS, et al: Effect on glycemic control of exenatide (synthetic exendin-4) additive to existing metformin and/or sulfonylurea
treatment in patients with type 2 diabetes. Diabetes Care 2003;26(8):23707.
3. FDA website. www.fda.gov/medwatch. Accessed June 2005.
4. Buse JB, et al. Effects of exenatide (exendin-4) on glycemic control over 30 weeks in sulfonylurea-treated patients with type 2 diabetes.
Diabetes Care 2004;27(11):262835.
5. Triplitt CL, et al: Diabetes Mellitus. In Dipro J, et al(eds). Pharmacotherapy: Pathophysiologic approach. 6th ed.
New York, NY: McGraw-Hill Companies. 2005;13331367
6. The American Association of Clinical Endocrinologists medical guidelines for the management of diabetes mellitus:
The AACE system of intensive diabetes self-management 2002 update. Endocrine Practice 2002;8(Suppl. 1):4065.
7. American Diabetes Association. Diagnosis and classification of diabetes mellitus. Diabetes Care 2005;28(Suppl. 1):S37S42.
8. American Diabetes Association. Standards of medical care in diabetes. Diabetes Care 2005;28(Suppl. 1):S4S36.
9. Poulsen MK, et al: The combined effect of triple therapy with rosiglitazone, metformin, and insulin aspart in type 2 diabetic patients.
Diabetes Care 2003;26(12):32733279.
10. Dailey GE, et al: Glycemic control with glyburide/metformin tablets in combination with rosiglitazone in patients with type 2 diabetes:
a randomized, double-blind trial. Am J Med 2004;116:223229.
11. DeFronzo RA, et al: Effects of exenatide (exendin-4) on glycemic control and weight over 30 weeks in metformin treated patients
with type 2 diabetes. Diabetes Care 2005;28(5):10921100.
12. Kendall DM, et al: Effects of exenatide (exendin-4) on glycemic control over 30 weeks in patients with type 2 diabetes treated with
metformin and a sulfonylurea. Diabetes Care 2005;28(5):10831091.
13. Klein S, et al: Weight management through lifestyle modification for the prevention and management of type 2 diabetes: rational
and strategies. Diabetes Care 2004;27(8):20672073.
14. Chobanian AV, et al: Seventh Report of the Joint National Committee on prevention, detection, evaluation and treatment of high
blood pressure. Hypertension 2003;42:12061252.
15. Heine RJ, et al: Exenatide versus insulin glargine in patients with suboptimally controlled type 2 diabetes. Annals of Internal Medicine
2005;143:559569.
16. Nathan DM, et al: Management of hyperglycemia in type 2 diabetes: A consensus algorithm for the initiation and adjustment of
therapy A consensus statement from the American Diabetes Association and the European Association for the Study of Diabetes.
Diabetes Care 2006;29(8)19631972.

32

American Diabetes Association Guidelines and Treatment Algorithms At-A-Glance

Appendix

American Diabetes Association


Self-Assessment Program
Educational Critiques

IN ORDER TO MAXIMIZE YOUR LEARNING EXPERIENCE IT IS IMPORTANT THAT YOU


DO NOT LOOK AT THE EDUCATIONAL CRITIQUES UNTIL AFTER YOU HAVE COMPLETED
MARKING YOUR ANSWERS FOR THE MCQs ON THE ANSWER SHEET.
DO NOT CHANGE THE ANSWERS MARKED ON YOUR ANSWER SHEET WHILE READING
THE CRITIQUES. THE INTENT OF ADA-SAP IS LEARNING NEW KNOWLEDGE AND
REINFORCING PREVIOUSLY LEARNED KNOWLEDGE. THERE IS NO PASS-FAIL SCORE.

Educational (Learning) Critiques

National Lipid Association Self-Study Module

33

Educational (Learning) Critiques


Items 175
Items 16
Answers 1 (A); 2 (B); 3 (C); 4 (C); 5 (D); 6 (C)
The ADA has established three criteria for the diagnosis of diabetes:
1. Symptoms of diabetes and a casual plasma glucose M200 mg/dL (11.1 mmol/L). Casual is defined as any time
without regard to time since last meal. The classic symptoms of diabetes include polyuria, polydipsia, and
unexplained weight loss, OR
2. Fasting plasma glucose (FPG) M126 mg/dL (7.0 mmol/L). Fasting is defined as no caloric intake for at least
8 hours, OR
3. 2-hour plasma glucose M200 mg/dL (11.1 mmol/L) during an oral glucose tolerance test (OGTT). The
test should be performed as described by the World Health Organization, using a glucose load containing
the equivalent of 75 g anhydrous glucose dissolved in water.
Impaired fasting glucose (IFG) is defined as FPG of 100125 mg/dL. Impaired glucose tolerance (IGT) is a
2-hour plasma glucose of 140199 mg/dL after a 75 g OGTT. Although the OGTT is more sensitive and modestly
more specific than FPG to diagnose diabetes, it is poorly reproducible and rarely performed in practice. Because
of ease of use, acceptability to patients, and lower cost, the FPG is the preferred diagnostic test. It should be noted
that the vast majority of people who meet diagnostic criteria for diabetes by OGTT, but not by FPG, will have
an A1C value l7.0%. The use of the A1C for the diagnosis of diabetes is not recommended by the ADA at this
time. Pre-diabetes is defined as having either IFG or IGT.
Bibliography
1. American Diabetes Association. Standards of medical care in diabetes 2007. Diabetes Care 2007;30(Supp1).

Item 7
Answer E
According to the ADA guidelines:
1. Testing for diabetes should be considered in all individuals at age 45 years and above, particularly in those
with a BMI M25 kg/m2 and, if normal, should be repeated at 3-year intervals.
2. Testing should be considered at a younger age or be carried out more frequently in individuals who are
overweight (BMI M25 kg/m2*) and have additional risk factors:
K Are habitually physically inactive.
K Have a first-degree relative with diabetes.
K Are members of a high-risk ethnic population (e.g., African-American, Latino, Native American,
Asian American, Pacific Islander).
K Have delivered a baby weighing L9 lb or have been diagnosed with GDM.
K Are hypertensive (blood pressure, M140/90 mm Hg).
K Have an HDL cholesterol level l35 mg/dL (0.90 mmol/L) and/or a triglyceride level L250 mg/dL
(2.82 mmol/L).
K Have polycystic ovary syndrome (PCOS).
K On previous testing, had IGT or IFG.
K Have other clinical conditions associated with insulin resistance (e.g., PCOS or acanthosis nigricans).
K Have a history of vascular disease.
* May not be correct for all ethnic groups.
34

American Diabetes Association Self-Assessment Program

Educational (Learning) Critiques

This 35-year-old African-American man has at least two CHD risk factors (positive family history of CVD and
hypertension) and, therefore, according to the NCEP ATP III guidelines, his LDL goal is l130 mg/dL. JNC7
also recommends drug therapy for a blood pressure of L130/85 mm Hg in patients with multiple risk factors.
The ADA does not recommend the routine use of OGTT to diagnose type 2 diabetes but does recommend the
testing of his overweight children for type 2 diabetes. The following is the criteria for testing for type 2 diabetes
in children:
K Overweight (BMI L85th percentile for age and sex, weight for height L85th percentile, or weight L120%
of ideal for height).
Plus any two of the following risk factors:
K Family history of type 2 diabetes in first- or second-degree relative.
K Race/ethnicity (Native American, African-American, Latino, Asian American, Pacific Islander).
K Signs of insulin resistance or conditions associated with insulin resistance (acanthosis nigricans, hypertension,
dyslipidemia, or PCOS).
K Maternal history of diabetes or GDM.
Age of initiation: age 10 years or at onset of puberty, if puberty occurs at a younger age.
Frequency: every 2 years.
Test: FPG preferred.
Clinical judgment should be used to test for type 2 diabetes in high-risk patients who do not meet these criteria.
Bibliography
1. American Diabetes Association. Standards of medical care in diabetes 2007. Diabetes Care 2007;30(Supp1).

Item 8
Answer C
The ADA states that a graded exercise test with electrocardiogram (ECG) monitoring should be seriously
considered before undertaking aerobic physical activity with intensity exceeding the demands of everyday living
(more intense than brisk walking) in previously sedentary patients with diabetes whose 10-year risk of a coronary
event is likely to be M10%. A diet prescription that focuses on the foods the patient is used to eating should also
be implemented.
This patients 39-year-old sister does not have a 10-year risk of coronary event M10% (her risk is l5%) and,
therefore, a stress test is not recommended. In regard to treatment, the ADA recommends early intervention with
metformin in combination with lifestyle changes (Figure 1). The ADA does not recommend low-carbohydrate
diets (restricting to total carbohydrate (l130 g/day). The long-term effects of these diets are unknown, and
although such diets produce short-term weight loss, maintenance of weight loss is similar to that from low-fat diets
and the impact on CVD risk profile is uncertain. Monitoring carbohydrate, whether by carbohydrate counting,
exchanges, or experience-based estimation, remains a key strategy in achieving glycemic control. For individuals
with diabetes, the use of glycemic index and glycemic load may provide a modest additional benefit for glycemic
control over that observed when total carbohydrate is considered alone. Therefore, there is not sufficient evidence
to recommend use of glycemic index or glycemic load for prevention of diabetes, although foods high in fiber are
encouraged.

Educational (Learning) Critiques

American Diabetes Association Self-Assessment Program

35

Diagnosis
Lifestyle Intervention + Metformin
No
Add Basal Insulin
(Most effective)
A1C M7%

No

Intensify Insulin
No

A1C M7%

A1C M7%

Yes*

Add Sulfonylurea
(Least expensive)
No

Yes*

A1C M7%

Add Glitazone

Add Glitazone
(No hypoglycemia)
A1C M7%

No

Yes*

Add Basal Insulin


No

Yes*

Yes*

Add Sulfonylurea

A1C M7%

Yes*

Add Basal or Intensify Insulin


Intensive Insulin + Metformin +/- Glitazone
Figure 1. Algorithm for the metabolic management of type 2 diabetes. Reinforce lifestyle intervention at every visit.
* Check A1C every 3 months until l7% and then at least every 6 months.
Although three oral agents can be used, initiation and intensification of insulin therapy is preferred based on effectiveness and expense.

Sulfonylurea and glitazone are recommended as add-on therapy to metformin if the A1C is L7.0%. Even though
this patient has low HDL and glitazone has diabetic dyslipidemic benefits and sulfonylurea lowers glucose rapidly,
the primary initial therapy should be metformin due to cost considerations, the absence of hypoglycemia, and
evidence of benefit.
Bibliography
1. American Diabetes Association. Standards of medical care in diabetes 2007. Diabetes Care 2007;30(Supp1).

Item 9
Answer A
According to the ADA guidelines:
K Patients with a systolic blood pressure of 130139 mm Hg or a diastolic blood pressure of 8089 mm Hg
should be given lifestyle and behavioral therapy alone for a maximum of 3 months and then, if targets are not
achieved, in addition, be treated with pharmacological agents that block the rennin-angiotensin system.
K Initial drug therapy for those with a blood pressure L140/90 mm Hg should be with a drug class demonstrated
to reduce CVD events in patients with diabetes (ACE inhibitors, angiotensin receptor blockers [ARBs]
-blockers, diuretics, and calcium channel blockers).
K All patients with diabetes and hypertension should be treated with a regimen that includes either an ACE
inhibitor or an ARB. If one class is not tolerated, the other should be substituted. If needed to achieve blood
pressure targets, a thiazide diuretic should be added.
K In individuals without overt CVD:
k The primary goal is an LDL l100 mg/dL (2.6 mmol/L).
k For those over the age of 40 years, statin therapy to achieve an LDL reduction of 3040% regardless
of baseline LDL levels is recommended.
k For those under the age of 40 years but at increased risk due to other cardiovascular risk factors who do not
achieve lipid goals with lifestyle modifications alone, the addition of pharmacological therapy is appropriate.
K Other goals include:
k Lower triglycerides to l150 mg/dL (1.7 mmol/L) and raise HDL cholesterol to L40 mg/dL (1.0 mmol/L).
In women, an HDL goal 10 mg/dL higher (L50 g/dL) should be considered.
36

American Diabetes Association Self-Assessment Program

Educational (Learning) Critiques

The primary treatment for dyslipidemia is a statin, unless the triglycerides are L500 mg/dL.
Bibliography
1. American Diabetes Association. Standards of medical care in diabetes 2007. Diabetes Care 2007;30(Supp1).

Item 10
Answer E
The ADA recommends all of the following ancillary tests as part of the initial workup of patients with newlydiagnosed type 2 diabetes. Urinary evaluation for microalbuminuria is important to assess early renal damage and
potentially increase the aggressiveness of blood pressure control. Patients with type 2 diabetes should also undergo
a dilated comprehensive eye exam by an ophthalmologist or optometrist as well as screening for distal symmetric
polyneuropathy using simple clinical tests (i.e., assess vibratory response in both feet). A comprehensive foot exam
is needed to evaluate for potential ulcers or loss of sensation with the appropriate referral for special footwear or
podiatric care.
Bibliography
1. American Diabetes Association. Standards of medical care in diabetes 2007. Diabetes Care 2007;30(Supp1):S4S41.
2. Nathan DM, et al: Management of hyperglycemia in type 2 diabetes: A consensus algorithm for the initiation and adjustment of therapy:
a consensus statement from the American Diabetes Association and the European Association for the Study of Diabetes. Diabetes Care
2006;29:19631972.

Item 11
Answer A
The ADA website has a risk calculator that can be a very valuable educational tool to assist clinicians to assess risk
of diabetes. The website link is www.diabetes.org/diabetesphd. This website is very patient-friendly and can help
educate regarding the benefits of weight reduction and other factors to reduce the risk of diabetes, heart disease,
stroke, and kidney disease. The Diabetes PhD risk calculator can also powerfully demonstrate the benefits of even
modest weight loss in reducing the risk of type 2 diabetes. In this patient, a 10% weight loss (20 lbs) would reduce
her risk of developing type 2 diabetes to less than 10%. Weight loss is the most powerful modifier for the prevention
of type 2 diabetes.
In the Diabetes Prevention Program (DPP), the 3234 enrolled subjects were slightly younger (mean age 51 years)
and more obese (mean BMI 34 kg/m2) but had nearly identical glucose intolerance compared with subjects in
the Finnish study. About 45% of the participants were from minority groups (e.g., African-American, Hispanic),
and 20% were M60 years of age. Subjects were randomized to one of three intervention groups, which included
the intensive nutrition and exercise counseling (lifestyle) group or either of two masked medication treatment
groups: the biguanide metformin group or the placebo group. The latter interventions were combined with
standard diet and exercise recommendations. After an average follow-up of 2.8 years (range 1.84.6 years), a 58%
relative reduction in the progression to diabetes was observed in the lifestyle group (absolute incidence 4.8%), and
a 31% relative reduction in the progression of diabetes was observed in the metformin group (absolute incidence
7.8%) compared with control subjects (absolute incidence 11.0%). On average, 50% of the lifestyle group achieved
the goal of M7% weight reduction, and 74% maintained at least 150 min/week of moderately intense activity.
No serious side-effects were seen in any group.
Bibliography
1. Diabetes Prevention Research Group: Reduction in the evidence of type 2 diabetes with life-style intervention or metformin.
N Engl J Med 2002;346:393403.
2. The Diabetes Prevention Program Research Group: The Diabetes Prevention Program: baseline characteristics of the randomized
cohort. Diabetes Care 2000;23:16191629.

Educational (Learning) Critiques

American Diabetes Association Self-Assessment Program

37

Item 12
Answer D
The ADA recently published a consensus statement of the implications for care in patients with pre-diabetes
(either impaired fasting glucose and/or impaired glucose tolerance). The ADA Panel recommended, based on
clinical trial evidence and cost considerations, that only metformin be considered as drug therapy for individuals
with IFG/IGT. In the DPP study, the subsets of the study cohort that had substantially increased benefit from
metformin were those participants l60 years of age and those who had a BMI M35 kg/m2. Therefore, the Panel
also recommends that metformin be limited to such individuals. Since individuals with associated risk factors for
diabetes, e.g., family history in first-degree relatives, elevated triglycerides, low HDL cholesterol, and hypertension,
are more likely to progress to diabetes, the presence of one or more of these factors may contribute to the decision
to treat with metformin. In addition, to better target a population likely to benefit from metformin therapy, an
unpublished analysis of data from the DPP study suggests that an A1C M6.0% approximately doubles the rate
of progression to diabetes in an IFG/IGT population.
A summary of these recommendations in shown in Table 1. Future recommendations may include other
medications if they prove to be effective, have a good safety profile, are tolerable, and are of relatively low cost.
Table 1. Treatment recommendation for individuals with IFG, IGT, or both.
Population

Treatment

IFG or IGT

Lifestyle modification (i.e., 510% weight loss and moderate


intensity physical activity p30 min/day)

Individuals with IFG and IGT and any of the following:


K L60 years of age.
K BMI M35 kg/m2.
K Family history of diabetes in first-degree relatives.
K Elevated triglycerides.
K Reduced HDL-C.
K Hypertension.
K A1C L6.0%.

Lifestyle modification (as above) and/or metformin*

* Metformin 850 mg b.i.d.

Bibliography
1. Nathan DM, et al: Impaired fasting glucose and impaired glucose tolerance. Diabetes Care 2007;30:753759.

Item 13
Answer B
The ADAs treatment algorithm for hyperglycemia strongly encourages the initiation of metformin in conjunction
with lifestyle changes. Therefore, although this patient has made some progress with weight loss, metformin
treatment should be initiated. Sulfonylureas are second-line agents and can effectively lower blood glucose acutely
in symptomatic individuals, but this patients hyperglycemia is not severe, and she is asymptomatic. Although
she has dyslipidemia, the ADA recommends TZDs as second-line agents but should be considered in patients
with low HDL/elevated triglycerides. The hyperglycemic management guidelines were released prior to the
approval of sitagliptin, which is a category B drug for use in women of child-bearing potential, but metformin
is still the first-line agent.
Bibliography
1. Nathan DM, et al: Management of hyperglycemia in type 2 diabetes: A consensus algorithm for the initiation and adjustment of therapy:
a consensus statement from the American Diabetes Association and the European Association for the Study of Diabetes. Diabetes Care
2006;29:19631972.

38

American Diabetes Association Self-Assessment Program

Educational (Learning) Critiques

Items 1418
Answers 14 (D); 15 (B); 16 (A); 17 (E); 18 (C)
Thiazolidinediones are contraindicated in patients with NYHA Class III/IV cardiac failure because of concerns
that they may precipitate pulmonary edema, but there is no evidence that they are harmful in Class I heart failure.
Rosiglitazone has recently been implicated as increasing the incidence of myocardial infarction and a borderline
significant increase in death in a meta-analysis. Thiazolidinediones also cause a modest reduction in the hematocrit
probably by a hemdilution effect. Gemfibrozil, but not fenofibrate, inhibits glucuronidation of statins. This is a
pathway that had not been recognized earlier as being important in statin catabolism. As a consequence, statin
levels have been demonstrated to be increased in subjects receiving gemfibrozil. Fenofibrate is extensively metabolized in the kidney, and therefore should be dose-adjusted in patients with moderate-to-severe renal impairment.
Gemfibrozil requires dose adjustment only in patients with severe renal impairment. In the clinical scenario
described in Item 17 the thiazolidinediones and fibrate are not contraindicated since the liver enzymes are less than
twice elevated, and these drugs may improve fatty liver changes.
Bibliography
1. Nesto RW, et al: American Heart Association; American Diabetes Association. Thiazolidinedione use, fluid retention, and congestive
heart failure: a consensus statement from the American Heart Association and American Diabetes Association. Circulation
2003;108(23):29418.
2. Lebovitz HE. Differentiating members of the thiazolidinedione class: a focus on safety. Diabetes Metab Res Rev 2002;18(Suppl 2):S239.
3. Prueksaritanont T, et al: Effects of fibrates on metabolism of statins in human hepatocytes. Drug Metab Dispos 2002;30(11):12807.
4. Gajdos M, et al: Fibrates and renal function. Clin Nephrol 2003;60(1):656.
5. On-Line: N Engl J Med 5/20/07.

Items 1923
Answers 19 (A); 20 (A); 21 (C); 22 (D); 23 (E)
The importance of adipocytokines to the genesis of insulin resistance and atherosclerosis is assuming increasing
importance. Among these, adiponectin has been shown to both enhance insulin sensitivity and inhibit monocyte
adhesion and infiltration of the vascular wall. By contrast, free fatty acids cause insulin resistance in part by interfering with hepatic insulin degradation causing hyperinsulinemia which leads to down-regulation of insulin
receptors and insulin resistance. TNF- increases insulin resistance by interfering with insulin-signaling pathways.
PAI-1 localizes in plaque where it may promote coagulation.
Bibliography
1. Molecular mechanisms of insulin resistance and the role of the adipocyte: Int J Obes Relat Metab Disord 2000;24(Suppl)4:S237.
2. Hoffstedt J, et al: Adipose tissue adiponectin production and adiponectin serum concentration in human obesity and insulin
resistance.J Clin Endocrinol Metab 2004;89(3):13916.
3. Hennes MM, et al: Effects of free fatty acids and glucose on splanchnic insulin dynamics. Diabetes 1997;46(1):5762.

Item 24
Answer B
The biguanides, represented by metformin in the United States, are effective oral hypoglycemic agents. They are
also effective when added to other oral hypoglycemic agents, such as thiazolidinediones or sulfonylureas, or when
added to insulin. They are usually dosed with meals. Use of biguanides has been associated with lactic acidosis
rarely with no obvious cause (50% death rate if it does occur) and especially in patients with renal insufficiency,
significant liver disease, or compromised LV function (90% death rates).
Bibliography
1. Mudaliar S, et al: The oral antidiabetic agents. In: Ellenberg and Rifkins Diabetes Mellitus, 6th Ed. Porte D, et al (eds).
McGraw-Hill, 2003;531564.

Educational (Learning) Critiques

American Diabetes Association Self-Assessment Program

39

Item 25
Answer B
Insulin resistance is associated with increased risk for cardiovascular disease. There have been multiple mechanisms suggested for this association. However, recently there has been increasing support for the pro-inflammatory
state that accompanies insulin resistance as being pathophysiologically important for accelerated large vessel
disease. Patients with insulin resistance sometimes have elevated homocysteine levels. However, the relationship
between homocysteine and the accelerated vascular disease of insulin resistance is less strong (therapy with folic
acid decreases homocysteine levels but does not reduce CV events). Patients with insulin resistance have increased
plasminogen activator inhibitor levels and, therefore, a pro-thrombotic state. While patients with insulin resistance
also have elevated insulin levels, the association between insulin levels and vascular disease is most likely related
to insulin resistance and not to direct toxic effect of insulin on the vasculature.
Bibliography
1. Haffner S, et al: Insulin-resistant prediabetic subjects have more atherogenic risk factors than insulin-sensitive prediabetic subjects.
Implications for preventing coronary heart disease during the prediabetic state. Circulation 2000;101:975980.
2. Hunt K, et al: Elevated carotid artery intima-media thickness levels in individuals who subsequently develop Type 2 diabetes.
Arterioscler Thromb Vasc Biol 2003;23:18451850.
3. Soinio M, et al: Elevated plasma homocysteine level is an independent predictor of coronary heart disease events in patients with
Type 2 diabetes mellitus. Ann Intern Med 2004;140:94100.

Item 26
Answer D
The ability to monitor blood glucose at home has revolutionized management of patients with diabetes. Blood
glucose levels are of value to physicians, and can be obtained either several times per week or several times per day,
depending on the clinical circumstance. While patients on insulin require more frequent monitoring patients with
type 2 diabetes on oral hypoglycemic therapy will also benefit from home monitoring. For patients on insulin
therapy, measuring the fasting blood glucose is the most reliable indicator for adjustment of the basal insulin dose.
In the most intensively controlled patients, it is useful to measure blood glucose before meals and at bedtime.
Bibliography
1. American Diabetes Association: Clinical Practice Recommendations 2004. Standards of medical care in diabetes.
Diabetes Care 27(Suppl. 1):2004;S15S35.

40

American Diabetes Association Self-Assessment Program

Educational (Learning) Critiques

Items 2731
Answers 27 (A); 28 (D); 29 (C); 30 (B); 31 (E)
There are now several classes of oral hypoglycemic agents available. Because these major classes have complementary mechanisms of action, they are often useful as combination therapy in individual patients. Sulfonylureas
work primarily by increasing insulin secretion from -cells. Thiazolidinediones are insulin sensitizers that increase
glucose disposal by increasing its uptake into muscle. The biguanides have a favorable effect on fasting blood glucose
by suppressing glucose production in the liver. The alpha-glucosidase inhibitors are not as efficacious for reducing
overall glycemia as sulfonylureas, thiazolidinediones or biguanides; however, they are useful for reducing postprandial blood glucose spikes, as they delay glucose uptake in the intestine and, thereby, inhibit glucose absorption.
Ingestion of food

GI tract
Sitagliptin
(DPP-4 inhibitor)

Release of
active incretins
GLP-1 and GIP
DPP-4
enzyme
Inactive Inactive
GLP-1
GIP

Glucose
dependent
qInsulin
(GLP-1
qGlucose
and GIP)
uptake by
Pancreas
peripheral
Beta cells
tissue
Alpha cells
Glucose
dependent
QGlucagon
(GLP-1)

QBlood glucose
in fasting and
postprandial states

QHepatic
glucose
production

Figure 2. Mechanism of action of sitagliptin.

The incretin hormones GLP-I and GIP are released by the intestine throughout the day, and levels are increased in
response to a meal. The incretins are part of an endogenous system involved in the physiologic regulation of glucose
homeostasis.
When blood glucose concentrations are normal or elevated, GLP-I and GIP increase insulin synthesis and release
from pancreatic -cells by intracellular signaling pathways involving cyclic AMP. With higher insulin levels, tissue
glucose uptake is enhanced.
In addition, GLP-I lowers glucagon secretion from pancreatic -cells.
K Decreased glucagon levels, along with higher insulin levels, lead to reduced hepatic glucose production
and are associated with a decrease in blood glucose levels in the fasting and post-prandial states.
K The beta-cell effects of GLP-I and GIP are glucose-dependent, but only the effect of GLP-1 on the alpha cell
is glucose-dependent. The glucose-dependency effect of GLP-1 on alpha- and beta-cells translate to a benefit
of rare hypoglycemia with incretin mimetics.
The activity of GLP-I and GIP is limited by the DPP-4 enzyme, which rapidly inactivates incretin hormones.
Concentrations of the active intact hormones are increased by sitagliptin, thereby increasing and prolonging the
action of these hormones. (Figure 2).
Bibliography
1. Mudaliar S, et al: The oral antidiabetic agents. In: Ellenberg and Rifkins Diabetes Mellitus, 6th Ed. Porte D, et al (eds). McGraw-Hill,
2003;531564.

Educational (Learning) Critiques

American Diabetes Association Self-Assessment Program

41

Items 3235
Answers 32 (B); 33 (C); 34 (A); 35 (D)
The oral hypoglycemic agents are generally safe; however, each major class is associated with specific serious sideeffects. Sulfonylureas, because they increase insulin secretion, are the class of oral hypoglycemic agents most closely
related to production of symptomatic hypoglycemia. The thiazolidinediones may increase fluid retention and
worsen peripheral edema. Their use in patients with symptomatic congestive heart failure is, therefore, not recommended. Biguanides most often give rise to gastrointestinal side-effects including mild nausea and some diarrhea,
but this can be minimized by titrating to a maximal effective dose of 2 g/day over 2 weeks. These side-effects are
generally self-limited. However, in patients with renal insufficiency or impaired cardiac function, biguanides have
been rarely associated with lactic acidosis (see Critique for Item 24). Alpha-glucosidase inhibitors decrease the
digestion of carbohydrates which are converted into gas and can cause bloating.
Bibliography
1. Mudaliar S, et al: The oral antidiabetic agents. In: Ellenberg and Rifkins Diabetes Mellitus, 6th Ed. Porte D, et al (eds).
McGraw-Hill, 2003;531564.

Item 36
Answer D
All of the options in this item have been shown to be beneficial as each can lead to weight loss. A reduction in
weight of as little as 4 kg has been shown to improve hyperglycemia. Whereas dietary and physical activity adjustments have been universally recommended and remarkably useful in some patients, weight loss following bariatric
surgery has been shown to be consistently effective in virtually eliminating diabetes if a mean weight loss of 20 kg
or more can be sustained. Although not appropriate for all patients, in properly-selected obese patients, bariatric
surgery can be an effective option. The results of recent studies have shown that bariatric surgery decreased
mortality rate in selected obese patients with type 2 diabetes. Studies of weight loss medications, such as orlistat
which reduces fat absorption and sibutramine which causes a modest decrease in appetite have demonstrated high
drop-out rates, significant side-effects, and cannot be recommended at this time for the treatment of diabetes.
Bibliography
1. Sjostrom L, et al: Swedish Obese Subjects Study Scientific Group. Lifestyle, diabetes, and cardiovascular risk factors 10 years after
bariatric surgery. N Engl J Med 2004;351:26832693.
2. Nathan DM, et al: Management of hyperglycemia in type 2 diabetes: A consensus statement from the American Diabetes Association
and the European Association for the Study of Diabetes. Diabetes Care 2006;29:19631972.

Item 37
Answer E
Exercise and weight loss have a beneficial effect on diabetes control but also on associated cardiovascular
risk factors, such as blood pressure and lipids. There is a psychological boost as well in exercising regularly
and seeing this improve your glycemic control and other risk factors. While lower weight and improved diabetic
control may slow the progression of neuropathy in the long-term, in the short-term the newly-exercising patient
with neuropathy is often at increased risk of foot trauma or ulceration. One should caution all patients with
diabetes, especially those with neuropathy, to look carefully at their feet everyday, particularly after exercise.
It is also important to have properly-fitting shoes and other equipment to lessen the possibility of injury.
Bibliography
1. Nathan DM, et al: Management of hyperglycemia in type 2 diabetes: A consensus statement from the American Diabetes Association
and the European Association for the Study of Diabetes. Diabetes Care 2006;29:19631972.
2. Diabetes Prevention Program Research Group: Impact of intensive lifestyle and metformin therapy on cardiovascular disease risk factors
in the Diabetes Prevention Program. Diabetes Care 2005;28:888894.

42

American Diabetes Association Self-Assessment Program

Educational (Learning) Critiques

Item 38
Answer C
Metformin is recommended as first line of treatment in most patients with newly-diagnosed diabetes that require
medication for all of the reasons mentioned in this item, except for option (C). While metformin is generally
well-tolerated it may have gastrointestinal side-effects, such as diarrhea or nausea, especially at higher doses.
More worrisome is the potentially fatal possibility of lactic acidosis which is actually very rare (l1 case per 100,000
treated patients). The risk of lactic acidosis can be reduced by avoiding metformin in patients with renal insufficiency (creatinine L1.4 mg/dL in women, L1.5 mg/dL in men), in association with the use of iodinated contrast
or with any condition that predisposes the patient to hypotension or hypoxia.
Bibliography
1. Nathan DM, et al: Management of hyperglycemia in type 2 diabetes: A consensus statement from the American Diabetes Association
and the European Association for the Study of Diabetes. Diabetes Care 2006;29:19631972.
2. Salpeter S, et al: Risk of fatal and nonfatal lactic acidosis with metformin use in type 2 diabetes mellitus. Cochrane Database Syst Rev
2006;CD002967.

Item 39
Answer C
The sulfonylurea medications increase insulin secretion which lowers blood glucose levels. This can cause
increased hunger, hypoglycemia and weight gain. It is common for a patient to gain about 2 kg when started
on a sulfonylurea, so they must be used with caution in those that are already overweight. Caution should also be
advised in the elderly that are more prone to hypoglycemia and drug interactions. Urinary tract infections (UTIs)
are common in patients with diabetes and sulfa drugs are commonly prescribed for UTIs. However, in one study
of over 900 elderly patients admitted to the hospital for hypoglycemia related to a taking a sulfonylurea, compared
with matched controls the patients with hypoglycemia were more than 6 times as likely to have been treated with
a sulfa antibiotic in the previous week. There is no logic to prescribe a sulfonylurea and glinide together as both
act on the same receptor on the beta-cell. Like almost any medication, the sulfonylureas can cause a rash.
However, a clear consistent adverse effect on cardiovascular risk has not been established. While the University
Group Diabetes Program suggested an increase in mortality when sulfonylureas are used in patients with type 2
diabetes, this was not confirmed by the UK Prospective Diabetes Study.
Bibliography
1. Juurlink DN, et al: Drug-drug interactions among elderly patients hospitalized for drug toxicity. JAMA 2003 Apr 2;289(13):16528.
PMID: 12672733.
2. Klimt CR, et al: The University Group Diabetes Program: a study of the effect of hypoglycemic agents on vascular
complications in patients with adult-onset diabetes. I. Design, methods and baseline characteristics. II. Mortality results.
Diabetes 1970;19(Suppl. 2):747830.
3. UK Prospective Diabetes Study (UKPDS) Group: Intensive blood glucose control with sulphonylureas or insulin compared
with conventional treatment and risk of complication in patients with type 2 diabetes (UKPDS 33). Lancet 1998;352:837853.

Educational (Learning) Critiques

American Diabetes Association Self-Assessment Program

43

Item 40
Answer C
The only significant drug-drug interaction with the glinides (e.g., repaglinide, nateglinide) concerns gemfibrozil,
which can lead to hypoglycemia by reducing the metabolism of either glinide drug by the liver. Therefore, when
hypertriglyceridemia and diabetes coexist, which often occurs, caution must be exercised in choosing therapy.
A method other than gemfibrozil, such as control of diabetes, dietary measures, fenofibrate or omega-3 fatty acids
should be used if triglyceride-lowering is needed.
Bibliography
1. Hasslacher C. Multinational Repaglinide Renal Study Group. Safety and efficacy of repaglinide in type 2 diabetic patients with and
without impaired renal function. Diabetes Care 2003 Mar;26(3):88691.
2. Blickle JF. Meglitinide analogues: a review of clinical data focused on recent trials. Diabetes Metab 2006 Apr;32(2):11320.

Item 41
Answer A
Metformin is associated with an expected decrease in A1C level of 1.5%. The only biguanide presently marketed
in the United States, metformin has been called an insulin sensitizer because it lowers blood glucose levels
without directly increasing insulin production or secretion by the pancreas. Its main mechanism however is
to reduce gluconeogenesis in the liver, thereby reducing glucose production. It likely exerts this effect through
alterations in AMP-kinase. Reductions in insulin resistance in the peripheral tissues have been observed but the
results are inconsistent.
Bibliography
1. Inzucchi SE, et al: Efficacy and metabolic effects of metformin and troglitazone in type II diabetes mellitus. N Engl J Med
1998;338:867872.
2. Hundal RS, et al: Mechanism by which metformin reduces glucose production in type 2 diabetes. Diabetes 2000;49:20632069.
3. Yu JG, et al: A comparison of troglitazone and metformin on insulin requirements in euglycemic intensively insulin-treated type 2
diabetic patients. Diabetes 1999;48:2414242.

Item 42
Answer D
Glitazones are associated with an expected decrease in A1C level of 0.51.4%. Pioglitazone and rosiglitazone are
the two thiazolidinediones (TZDs) that are currently available. The primary mechanism of action of the TZDs is to
enhance insulin sensitivity by improving the utilization of glucose by the peripheral adipocytes and skeletal muscle.
TZDs create this effect by stimulating a nuclear transcription factor called PPAR-. These receptors impact lipid
and carbohydrate metabolism by activating many (L1000), not just several, genes in various tissues. PPAR- is
most highly expressed in adipocytes and TZDs and are thought to act within fat cells to effect the production of
free fatty acids, adiponectin and TNF-, all of which impact insulin sensitivity. Some have described a reduction in
hepatic glucose production but not to the extent of metformin.
Bibliography
1. Frias JP, et al: Metabolic effects of troglitazone therapy in type 2 diabetic, obese, and lean normal subjects. Diabetes Care 2000;23: 6469.
2. Nolan JJ, et al: Improvement in glucose tolerance and insulin resistance in obese subjects treated with troglitazone. N Engl J Med
1994;331:11881193.

Item 43
Answer C
Alpha-glucosidase inhibitors (AGIs) are associated with an expected decrease in A1C level of 0.50.8%.
This drug class includes acarbose and miglitol. AGIs work by delaying the absorption of carbohydrates in the
gut. This has the most significant effect on the post-prandial blood glucose level so these medications are taken
with meals. The AGIs directly inhibit an enzyme that breaks down complex carbohydrates on the brush border of
the enterocyte. Unfortunately, the mechanism that creates their efficacy also yields their most significant side-effect
(intestinal bloating and gas). This is because the carbohydrates that are not absorbed are acted upon by the flora
of the gut to produce gas.
44

American Diabetes Association Self-Assessment Program

Educational (Learning) Critiques

Bibliography
1. Lebowitz HE. -glucosidase inhibitors as agents in the treatment of diabetes. Diabetes Rev 1998;6:132145.
2. Hanefeld M, et al: Therapeutic potentials of acarbose as first-line drug in NIDDM insufficiently treated with diet alone. Diabetes Care
1991;14 :732737.
3. Hotta N, et al: Long-term effect of acarbose on glycaemic control in non-insulin-dependent diabetes mellitus: a placebo-controlled
double-blind study. Diabet Med 1993;10:134138.

Item 44
Answer B
Glinides (also known as meglitinides) are associated with an expected decrease in A1C level of 11.5%. Two
glinides are available in the US at this time. Repaglinide is somewhat more efficacious, reducing A1C levels by
1.5%. Nateglinide when used as monotherapy reduces A1C by about 1%. Each of these works by acting upon the
same receptor as the sulfonylureas although at a different site. This causes a sequence of events that includes
closure of the ATP-dependent K channels which stimulate insulin secretion from the pancreas. These medications
are taken just prior to meals and act primarily on the post-prandial glucose level.
Bibliography
1. Scarsi M,et.al: Sulfonylureas and Glinides Exhibit PPAR- Activity: A Combined Virtual Screening and Biological Assay Approach.
Mol Pharmacol 2006 Nov 3.
2. Nathan DM, et al: Management of hyperglycemia in type 2 diabetes: A consensus statement from the American Diabetes Association
and the European Association for the Study of Diabetes. Diabetes Care 2006;29:19631972.

Item 45
Answer E
Sitagliptin is associated with an expected decrease in A1C level of 0.51.5%. Sitagliptin is a dipeptidyl peptidase-4
(DPP-IV) inhibitor. This acts upon the incretin system, which is a group of hormones active in the gut. One of
these hormones is GLP-I. DPP-IV inhibitors reduce the breakdown of many peptides, including GLP-I, thereby
increasing their activity. The main effect is to improve glucose-dependent insulin secretion and a glucosedependent suppression of inappropriate glucagon secretion. In addition, it results in an increased endogenous
GLP-1 effect of 24 times normal, which does not sufficiently suppress appetite or slow gastric emptying enough
to decrease weight. Though it is weight-neutral, it has no increased GI side-effects compared to placebo.
Bibliography
1. Nathan DM, et al: Management of hyperglycemia in type 2 diabetes: A consensus statement from the American Diabetes Association
and the European Association for the Study of Diabetes. Diabetes Care 2006;29:19631972.
2. Aschner P, et al: Effect of the dipeptidyl peptidase-4 inhibitor sitagliptin as monotherapy on glycemic control in patients with type 2
diabetes. Diabetes Care 2006 Dec;29(12):26327.

Item 46
Answer C
The only antihypertensive agents proven to prevent progression to microalbuminuria are the angiotensinconverting enzyme inhibitors (ACE-I). A recent review looked at the question of preventing nephropathy in
patients with type 1 or type 2 diabetes who had normal kidney function with or without hypertension. The absence
of nephropathy was defined as albumin excretion less than 30 mg/day on three serial measurements. Most of the
trials reviewed involved patients with type 2 diabetes with hypertension.
ACE-Is were the only agents that clearly showed evidence of preventing progression to microalbuminuria compared
to placebo or calcium channel blockers. The number of patients with diabetes one needed to treat to prevent
progression of one patient to microalbuminuria was 25. ACE-Is did not reduce progression to end-stage renal
disease or all-cause mortality. There were not enough studies to determine if normotensive patients received any
benefit.

Educational (Learning) Critiques

American Diabetes Association Self-Assessment Program

45

Angiotensin receptor blockers (ARBs) have also been shown to slow the progression of nephropathy associated
with diabetes. The summary of a very recent review of the effect of ACE-Is and ARBs is worth quoting in full.
Fifty studies (13215 patients) were identified. Thirty eight compared ACE-I with placebo, five compared ARBs
with placebo and seven compared ACE-Is and ARBs directly. There was no significant difference in the risk of
all-cause mortality for ACE-I versus placebo (RR 0.91, 95% CI 0.711.17) and ARBs versus placebo (RR 0.99,
95% CI 0.851.17). A subgroup analysis of studies using full-dose ACE-Is versus studies using half or less than
half the maximum tolerable dose of ACE-Is showed a significant reduction in the risk of all-cause mortality with
the use of full-dose ACE-I (RR 0.78, 95% CI 0.610.98). Baseline mortality rates were similar in the ACE-I and
ARB studies. The effects of ACE-Is and ARBs on renal outcomes (ESKD, doubling of creatinine, prevention of
progression of micro- to macroalbuminuria, remission of micro- to normoalbuminuria) were similarly beneficial.
Reliable estimates of effect of ACE-Is versus ARBs could not be obtained from the three studies in which they
were compared directly because of their small sample size.
Bibliography
1. Strippoli GFM, et al: Antihypertensive agents for preventing diabetic kidney disease. The Cochrane Database of Systematic Reviews
2006 Issue 4.
2. Strippoli GFM, et al: Angiotensin converting enzyme inhibitors and angiotensin II receptor antagonists for preventing the progression
of diabetic kidney disease. The Cochrane Database of Systematic Reviews 2006 Issue 4.

Item 47
Answer B
While nearly all of the published studies regarding the use of statins and prevention of cardiovascular disease event
rates have shown a reduction, the Prospective Study of Pravastatin in the Elderly at Risk (PROSPER) trial found
a trend toward harm regarding overall mortality. This trend did not reach the level of statistical significance.
In the PROSPER study 5804 people aged 7082 years were given pravastatin (40 mg/day; n = 2891) or placebo
(n = 2913). Follow-up was 3.2 years on average and our primary endpoint was a composite of coronary death,
non-fatal myocardial infarction, and fatal or non-fatal stroke. Pravastatin lowered LDL cholesterol concentrations
by 34% and reduced the incidence of the primary endpoints (hazard ratio 0.85, 95% CI 0.740.97, P = 0.014).
Coronary heart disease death and non-fatal myocardial infarction risk were also reduced (0.81, 0.690.94,
P = 0.006). Stroke risk was unaffected (1.03, 0.811.31, P = 0.8), but the hazard ratio for transient ischaemic
attack was 0.75 (0.551.00, P = 0.051). New cancer diagnoses were more frequent on pravastatin than on placebo
(1.25, 1.041.51, P = 0.020). However, a meta-analysis of five studies found no association between statin use
and the risk of non-fatal and fatal cancers. All the other options in this item are correct.
Bibliography
1. Shepherd J, et al: Pravastatin in elderly individuals at risk of vascular disease (PROSPER): A randomised controlled trial.
Lancet 2002;360:162330.

Item 48
Answer D
Alpha-glucosidase inhibitors (AGIs), such as acarbose, work by delaying the intestinal absorption of glucose. They
act primarily on post-prandial blood glucose levels and are unlikely to cause hypoglycemia. They generally have to
be dosed with each meal and the dose of acarbose is 50 mg t.i.d. The advantage of this class of drugs is they are
unlikely to cause any systemic effects and are useful in patients with kidney or liver insufficiency where other drugs
may be contraindicated. Unfortunately, their use is limited by side-effects primarily gastrointestinal in that the
undigested sugars are acted upon by intestinal bacteria to produce gas. Therefore patients with low tolerance for
side-effects would not be good candidates for this class of medications.
The AGIs are less potent than other classes of medications in reducing A1C from 0.50.8% so they are not the first
choice for monotherapy if a large drop in A1C is needed. Thus a patient poorly-controlled with a low tolerance for
medication would be the least suitable candidate for this class of medications. A recent review noted that there is no
evidence that AGIs reduce morbidity or mortality.
46

American Diabetes Association Self-Assessment Program

Educational (Learning) Critiques

Bibliography
1. Nathan DM, et al: Management of hyperglycemia in type 2 diabetes: A consensus statement from the American Diabetes Association
and the European Association for the Study of Diabetes Mellitus, Diabetes Care 2006;29:19631972.
2. Van de Laar FA, et al: Alpha-glucosidase inhibitors for type 2 diabetes. The Cochrane Database of Systematic Reviews 2006 Issue 4.

Item 49
Answer C
Each of the options listed (A, B, D, E), except option (C), is associated with significant insulin deficiency. Whether
the fasting blood glucose level is high (L250 mg/dL), the A1C is L10%, or the patient is symptomatic with
polyuria and polydypsia, the quickest way to stabilize the patient and reduce their blood glucose is by starting
insulin. When ketonuria is present it also indicates that the peripheral tissues are unable to utilize glucose as a fuel
and must resort to lipolysis. Often insulin can be started, quickly titrated to optimum levels and oral agents can be
added. As the blood glucose level corrects, eventually the insulin can often be withdrawn. The initiation of insulin
often is associated with a weight gain of 24 kg as the glycosuria is arrested and these calories are no longer lost.
However, the specific body mass index should not be a criterion for deciding on the initiation of insulin therapy.
Bibliography
1. Nathan DM, et al: Management of hyperglycemia in type 2 diabetes: A consensus statement from the American Diabetes Association
and the European Association for the Study of Diabetes Mellitus. Diabetes Care 2006;29:19631972.

Item 50
Answer A
The American Diabetes Association (ADA) and the European Association for the Study of Diabetes (EASD)
recently issued a consensus statement regarding the management of hyperglycemia in patients with type 2 diabetes.
In this statement the goals and principles of therapy were outlined and the various options for treatment discussed.
There are many factors to consider when lifestyle modifications are inadequate to control the blood glucose level and
the decision to start a patient on medication has been made. Considerations include the goals of the patient, the anticipated short- and long-term benefits of therapy, and the short- and long-term risks, possible side-effects, expected
adherence to therapy, cost of the medication, insurance coverage and non-glycemic effects of the medication.
The patient described in this item is a common one seen in clinical practice. She is overweight, hypertensive and
requires a significant reduction in her A1C level to be at her goal. Metformin is an excellent initial choice for her
starting medication as it is generally well tolerated, has a low incidence of hypoglycemia, and generally does not lead
to additional weight gain. It is also available in a generic formulation, and is relatively effective in controlling blood
glucose.
A sulfonylurea, while inexpensive, is more likely to cause hypoglycemia and additional weight gain. A glitazone has
the advantage of possibly improving the patients lipid profile, but may cause water retention or weight gain and
is much more expensive than either metformin or a sulfonylurea. Inhaled insulin is relatively new to the US market,
is expensive and would not be considered as a first line medication in most patients. Sitagliptin, a DPP-IV inhibitor
which acts upon the incretin system to increase insulin release, is also new to the market, expensive and too novel to
recommend as starting therapy in most patients. For these reasons and others, the consensus panel recommended
metformin as the initial choice of drug in most situations. Metformin is recommended as the initial pharmacologic
therapy, in the absence of specific contraindications, for its effect on glycemia, absence of weight gain or hypoglycemia, generally low level of side-effects, high level of acceptance, and relatively low cost. Table 2 summarizes
the effects of multiple treatment options in diabetes.

Educational (Learning) Critiques

American Diabetes Association Self-Assessment Program

47

Table 2. Summary of anti-diabetic interventions as monotherapy.


Expected decrease
in A1C (%)
Advantages

Disadvantages

Lifestyle to
decrease weight and
increase activity

12

Low cost, many benefits

Fails for most in first year

Metformin

1.5

Weight neutral, inexpensive

GI side-effects, rare lactic acidosis

Interventions
Step 1: Initial Therapy

Step 2: Additional Therapy


Insulin

1.52.5

No dose limit,inexpensive,
improved lipid profile

Injections, monitoring,
hypoglycemia, weight gain.

Sulfonylureas

1.5

Inexpensive

Weight gain, hypoglycemia*

TZDs

0.51.4

Improved lipid profile

Fluid retention, weight gain,


expensive

-Glucosidase inhibitors

0.50.8

Weight neutral

Frequent GI side-effects,
t.i.d. dosing, expensive.

Exenatide

0.51.0

Weight loss

Injections, frequent GI side-effects,


expensive, little experience

Glinides

11.5

Short duration

t.i.d. dosing, expensive

Pramlintide

0.51.0

Weight loss

Injections, t.i.d. dosing,


frequent GI side-effects,
expensive, little experience

Other Drugs

* Severe hypoglycemia is relatively infrequent with sulfonylurea therapy. The longer-acting agents (e.g., chlorpropamide, glyburide [glibenclamide], and
sustained-release glipizide) are more likely to cause hypoglycemia than glipizide, glimepiride, and gliclazide. Repaglinide is more effective at lowering
A1C than nateglinide.

Bibliography
1. Nathan DM, et al: Management of hyperglycemia in type 2 diabetes: A consensus statement from the American Diabetes Association
and the European Association for the Study of Diabetes Mellitus. Diabetes Care 2006;29:19631972.

Item 51
Answer A
The consensus statement of the ADA and EASD recommends an algorithm for the introduction of insulin
if that is deemed necessary (Figure 3). After allowing for patient characteristics, lifestyle, and meal schedules,
it is recommended to start with 10 units or 0.2 units/kg of bedtime intermediate-acting insulin or a similar dose
of long-acting insulin taken at bedtime or in the morning. A single dose of insulin is likely to have better compliance
and by starting low and titrating up, the chances of hypoglycemia are minimized. Blood glucose levels should be
monitored at least daily and the dose adjusted frequently (typically by 2 units every 3 days) until a desired fasting
level of blood glucose is obtained (70130 mg/dL). If the fasting glucose is L180 mg/dL, a faster titration may
be initiated by increasing the dose by 4 units every 3 days.

48

American Diabetes Association Self-Assessment Program

Educational (Learning) Critiques

Start with bedtime intermediate-acting insulin


or bedtime or morning long-acting insulin.
Can initiate with 10 units or 0.2 units per kg
kcehC
rcni

fats

,esod ae
i era

g ni
yt yl acip
gnar te

(esocul f

gni itsre
yb 2 nu ve sti
(70
e 31 0 /gm d
ral i reg
mercn
ulg esoc
L180

grat n
rcn sae ni esod
3d ya ,s fi fagnits

i nac

M7%afet 2 r 3m tno
NO YES

A1C
Ifh yplgo
fro a
its culg n
(3.8 9m /lom L),er
M4un sti ,1 ro

nit oC

er u
kceh
ve yre

ycime
ruc o a
,s
el so
vel L70 /gm Ld
cu d eb
itd od em
es yb
0 %if sod e L60inu
st

;nemig
A1C
3mno

nar
sht

Prl-e cnu
eg :ad
rb ta nilus

)kc su l au yd ylia dna


yre 3d ya s tnu fli tsa gni vel sle
Lo 3r 8. 9 7.2 m 2 /lom
L);
stne
,.e b .g y4 stinu
ve yre
/gm
Ld ( L1m 0
/lom
L)

tuo gb h
par

sh ?

Iffa its gb n
t ni
3.8 97. m 2
lom
dna
,debped
idne
ni jetc ;noi nac
dna
da juts yb u 2 tin
ro

fo
itc gn
fats

a-di
kae

Prid-e

ren

gnar

e :ad
rb ta
par

NO

ca-di
A1C

tegra
ar (egn
70
/L),hc kce gb l-erp
gn b no
r g use stl
su yl au
geb
ni htiw
e s ver y3 yad
us

uo gb
ot f
NPHsni ilu n
fkae
tsa ro
nit ta g ul hcn
M7%af ret

Prebgnar

31 0 /gm
Ld )
,hcnu
nidren ,
,da s d noce
d
~4u tin s
itn gb l ni nar eg

gb d
e :ad

sni

nilu

tuo
ar -dip
ta n id

fo
tca

gni
re

YES
Rehc
fo tu

nar

,eg am
fi A1c C

2 kceh
p-er

kce

laem- rp
gb vel sle na fi d
yn e t d da o
ona d
eht i r n jetc
tno uni se ot o eb
tu fo ar ,egn
p h- tso narplaid vel sle na da
juts
dnar
lai ipar a-d itc i gn lusn
ni

;noi

Figure 3. Algorithm for starting insulin.


Initiation and adjustment of insulin regimens. Insulin regimens should be designed taking lifestyle and meal schedule into account. The algorithm can only
provide basic guidelines for initiation and adjustment of insulin. Premixed insulins are not recommended during adjustment of doses; however, they can be used
conveniently, usually before breakfast and/or dinner if proportion of rapid- and intermediate-acting insulins is similar to the fixed proportions available.
bg = blood glucose.

Bibliography
1. Nathan DM, et al: Management of hyperglycemia in type 2 diabetes: A consensus statement from the American Diabetes Association
and the European Association for the Study of Diabetes Mellitus. Diabetes Care 2006;29:19631972.

Educational (Learning) Critiques

American Diabetes Association Self-Assessment Program

49

Item 52
Answer B
Since secretogogues, such as sulfonylureas and glinides, increase insulin secretion, they should be discontinued if
short-acting insulin is added to the patients regimen. Otherwise, there is a high risk of hypoglycemia and it is more
difficult to titrate the correct dose of the short-acting insulin. It is also expensive and unnecessary to continue both
types of therapy. The basal insulin should be continued however in that the short-acting insulin will primarily affect
the post-prandial blood glucose level and be inadequate to control the blood glucose level during fasting periods,
such as sleep. ACE-Is, while useful in preserving renal function, have no direct bearing on the initiation of shortacting insulins. Carbohydrate intake should remain constant; otherwise the risk of hypoglycemia will be increased.
Bibliography
1. Nathan DM, et al.: Management of hyperglycemia in type 2 diabetes: A consensus statement from the American Diabetes Association
and the European Association for the Study of Diabetes Mellitus. Diabetes Care 2006;29:19631972.

Item 53
Answer D
All the listed options (A, B, C, E) except option (D) (increasing the metformin to 1000 mg t.i.d.) are acceptable
alternatives. The maximum recommended daily dose of metformin is 2550 mg. The maximum recommended dose
of the extended-release form of metformin is 2000 mg. In the recent consensus panel of the ADA and EASD, there
was no clear preferred choice for the second agent to add in this situation. A glitazone, sulfonylurea or a single dose
of basal or intermediate-acting insulin are all acceptable options. The choice of insulin is favored if the A1C is
L8.5% or the patient is symptomatic from the elevated blood glucose. Other factors to consider include the cost
of the medication and patient preferences. If 2 oral medications at maximum dosage do not adequately control the
patients blood glucose, leading to a A1C of l7.0%, the next step would be to start insulin therapy. The general
consensus is that since it is the most effective therapy for achieving the recommended goal, the institution of insulin
should occur earlier in the therapeutic regimen than it does with many patients. If the idea is introduced early in the
process and encouraged with the proper support, most patients will accept the use of insulin and be pleased by its
effectiveness in controlling their diabetes.
Bibliography
1. Nathan DM, et al: Management of hyperglycemia in type 2 diabetes: A consensus statement from the American Diabetes Association
and the European Association for the Study of Diabetes Mellitus. Diabetes Care 2006;29:19631972.

Item 54
Answer E
Although the UKPDS did not show a statisically significant difference in macrovascular outcomes with intensive vs.
conventional therapy for diabetes, epidemiologic analysis of study data showed significantly lower CVD rates with
lower achieved A1C levels, as described in options AD. The reductions were incremental and the lower the A1C
was maintained the lower the complication rate as described in options AD. The rate for heart failure was actually
only 16% reduction/1% drop in A1C as noted in Figure 4, much less than 42%.

50

American Diabetes Association Self-Assessment Program

Educational (Learning) Critiques

Fatal and non-fatal myocardial infarction

10

Fatal and non-fatal stroke

P l0.0001

Hazard ratio

P = 0.035

12% decrease per


1% reduction in A1C

14% decrease per


1% reduction in A1C

0.5
Microvascular end points

10

Cataract extraction

P l0.0001

Hazard ratio

P l0.0001

19% decrease per


1% reduction in A1C

37% decrease per


1% reduction in A1C

0.5
Amputation or death from
peripheral vascular disease

10

Heart failure

P l0.0001

Hazard ratio

P = 0.021

1
0.5

43% decrease per


1% reduction in A1C
5

16% decrease per


1% reduction in A1C
10 5

10

Updated mean haemoglobin A1C concentration (%)


Figure 4.
Hazard ratios, with 95% confidence intervals as floating absolute risks, as estimate of association between category of updated mean A1C concentration and
myocardial infarction, stroke, microvascular end points, cataract extraction, lower extremity amputation or fatal peripheral vascular disease, and heart failure.
Reference category (hazard ratio 1.0) is A1C l6% with log linear scales. P value reflects contribution of glycemia to multivariate model. Data adjusted for age at
diagnosis of diabetes, sex, ethnic group, smoking, presence of albuminuria, systolic blood pressure, high and low density lipoprotein cholesterol, and triglycerides.

Bibliography
1. Stratton IM, et al: Association of glycaemia with macrovascular and microvascular complications of type 2 diabetes (UKPDS 35):
prospective observational study. BMJ 2000;321:405412.

Item 55
Answer B
All patients with diabetes should receive yearly influenza vaccinations. Patients with diabetes less than 65-years-old
should receive a pneumococcal vaccine and it should be repeated once after the age of 65 if it has been L5 years
since the last vaccination. The other vaccines mentioned are indicated only in special populations at increased risk.
The rates of vaccination can be increased by using screening tools and standing order protocols developed by the
Centers for Disease Control.
Educational (Learning) Critiques

American Diabetes Association Self-Assessment Program

51

Bibliography
1. www.cdc.gov/nip/recs/adult-schedule.htm
2. www.immunize.org/catg.d/4036need.pdf (screening tool)
3. www.immunize.org/catg.d/p3075.pdf (standing orders for Pneumococcal vaccine)

Item 56
Answer C
The UK Prospective Diabetes Study (UKPDS) Group found that while type 2 diabetes is a progressive disease,
on average, patients had already lost about half (50%) of their beta-cell function by the time they were diagnosed.
This highlights the need to intervene with patients before frank diabetes is confirmed. Multiple interventions
including diet, exercise, weight loss and certain medications have been found to reduce the progression from
glucose intolerance to diabetes.
Bibliography
1. UK Prospective Diabetes Study Group. UK Prospective Diabetes Study 16. Overview of 6 years therapy of type 2 diabetes:
a progressive disease. Diabetes 1995;44:12491258.

Item 57
Answer D
Hypoglycemia generally occurs gradually and is preceded by warning signs, such as anxiety, diaphoresis,
tachycardia and shakiness. The level of serum glucose that causes symptoms of hypoglycemia may vary from
patient to patient and even within the same patient at times. Impaired level of consciousness may occur with
hypoglycemia but death is uncommon, unless warning signs are ignored for a prolonged period or a serious
medication error is made. Death from low blood glucose levels alone rarely occurs unless it involves a dangerous
situation, e.g., while driving, climbing stairs, etc.
A recent study of adverse drug reactions that led to an emergency department visit showed that there are about
700,000 such patients visits annually. One quarter of these visits were by patients over the age of 65 and almost
3500 patients were admitted. The medications most commonly causing adverse events were insulins, oral hypoglycemics. opioid-containing products, anticoagulants, amoxicillin-containing products, and antihistamines/cold
products. One-third of adverse events in elderly patients were caused by warfarin, insulin, and digoxin. Over 85%
of elderly patients who presented with an unintentional overdose were taking anti-hyperglycemic agents, warfarin,
anti-convulsants, digoxin, theophylline, or lithium.
Bibliography
1. American Diabetes Association. Diabetes Care 2007;30(Suppl 1):S85.
2. Budnitz DS, et al: National surveillance of emergency department visits for outpatient adverse drug events. JAMA 2006;296:185866.

Item 58
Answer E
Insulin actually promotes the storage of triglycerides into adipose tissue, not the degradation of triglycerides into
FFAs. Therefore, if there is insulin resistance, the uptake of triglycerides is reduced and more FFAs are present in
the blood stream. All the other options (A, B, C, D) are consistent with the expected effects of insulin resistance.
The normal suppression of gluconeogenesis by insulin in the liver is impaired. The normal uptake of glucose by
skeletal muscle is reduced leading to elevated glucose levels. The normal glucose disposal, which includes glucose
oxidation and glycogen synthesis, by skeletal muscle, is impaired, raising the glucose levels further. Mildly elevated
levels of FFAs do not impair -cell function initially but actually increase insulin secretion to compensate for the
early phase of insulin resistance. As the FFA levels rise however, -cell dysfunction and damage occurs and the
compensatory ability to increase insulin production is overwhelmed and relative insulin deficiency occurs.
Bibliography
1. Unger RH. Lipotoxic diseases. Ann Rev Med 2002;53:319336.

52

American Diabetes Association Self-Assessment Program

Educational (Learning) Critiques

Item 59
Answer E
Environmental factors, such as diet and exercise, clearly impact the development of pancreatic -cell dysfunction.
Genetic factors, such as family history of diabetes or glucose intolerance increase the chances significantly of developing the same conditions. Type 2 diabetes is 26 times more prevalent in African-Americans, Native Americans,
Pima Indians, and Hispanic Americans in the US than in white Americans. In addition, 39% of patients with type 2
diabetes have at least one parent with the disease. As discussed in the critique for Item 60, FFAs contribute to
pancreatic -cell dysfunction by direct toxicity to the islet cells. Studies also suggest that glucose itself further
increases -cell loss by several mechanisms including protein glycation and glucokinase down-regulation, leading
to reduced production of insulin and even -cell death. There does not appear to be any direct insulin toxicity
which contributes to pancreatic -cell dysfunction.
Bibliography
1. Unger RH. Lipotoxic diseases. Ann Rev Med 2002;53:319336
2. Klein BE, et al: Parental history of diabetes in a population-based study. Diabetes Care 1996;19:827.
3. Harris MI, et al: Prevalence of diabetes, impaired fasting glucose, and impaired glucose tolerance in U.S adults: the Third National
Health and Nutrition Examination Survey, 19881994. Diabetes Care 1998;21:518.

Item 60
Answer C
Option (A) This is diabetic dermopathy which often occurs as pigmented, pretibial, symmetric pink and
brown-bronze atrophic shiny skin patches. In this picture we see the leg of a 59-year-old man with
diabetes. Although this condition is not specific to diabetes, it is probably the most common finding
in the skin of diabetics. These pigmented pretibial patches may be a marker for underlying cutaneous
microangiopathy.
Option (B) Acanthosis nigricans is a pigmenting disorder which causes velvety, light-brown-to-black markings
usually on the neck, under the arms, or in the groin. Acanthosis nigricans is most often associated with
obesity and diabetes. It is thought to be caused by elevated insulin levels during the early phase of
insulin resistance which activates insulin receptors in the skin, causing it to be more heavily pigmented.
Option (C) This is a case of scabies which is not specifically related to diabetes.
Option (D) This is a diabetic foot ulcer. These develop from a combination of neuropathy causing an underestimation of pressure on an area of the foot, and microangiopathy, poor blood flow and wound healing.
Bibliography
1. All of the images were borrowed with permission from:
Cohen BA, et al: DermAtlas. Johns Hopkins University, 2006. www.DermAtlas.org

Item 61
Answer E
Patients who develop type 2 diabetes mellitus have a progressive decline in -cell function. The UKPDS showed
that at the time of diagnosis of type 2 diabetes mellitus one has lost p50% of -cell function and continues to lose
-cell function over time (Figure 5). The graph thus suggests that loss of -cells may accrue for up to p10 years
prior to the diagnosis of type 2 diabetes mellitus. This estimate is confirmed by retinopathy rates that exist at time
of diagnosis of type 2 diabetes mellitus compared to time known to develop retinopathy in type 1 diabetes where
time of onset is more precisely defined (NHANES III). The question had been raised that if one screened for type
2 diabetes mellitus in patients at risk, before they have presumably lost p50% of their -cells, and found patients
with IGT, might one delay or prevent the onset of overt diabetes? Many studies, pictorialized in Figure 6, now have
shown reduced risk of developing type 2 diabetes mellitus with lifestyle changes, and with metformin in the DPP4
study, with acarbose in the Stop-NIDDM trial and with rosiglitazone in the DREAM trial. However in the DREAM
trial as well, ramipril was not shown to delay or prevent type 2 diabetes mellitus. The ADA recently recommended
a combination of lifestyle changes and metformin for patients with IFG and IGT.
Educational (Learning) Critiques

American Diabetes Association Self-Assessment Program

53

-cell function (% of normal by HOMA)

Time of diagnosis

100
?
80
60

Pancreatic function = 50% of normal

40
20
0
-10

-9

-8

-7

-6

-5

-4

-3

-2 -1
Years

Figure 5. Decline of -cell function in the UKPDS illustrates progressive nature of diabetes.

60

58%

62%

58%

55%

55%

50
42%

41%

40
31%
25%
PIOPOD

DREAM

XENDOS

TRIPOD

STOPNIDDM

DPP-Metformin

10

DPP-Lifestyle

20

Da Qing
Diet + Exercise

30

Finnish

Diabetes Mellitus Reduction (%)

70

0
Diabetes Prevention Clinical Trials
Figure 6. Is it possible to delay the onset of type 2 diabetes mellitus?

Bibliography
1. Wajchenberg, BL. -Cell Failure in Diabetes and Preservation by Clinical Treatment. Endocr Rev 2007;28(2):187218.
2. Gillies. Pharmacological and lifestyle interventions to prevent or delay type 2 diabetes in people with IGT. BMJ 2007;334:299302.
3.

Jeon C. Physical Activity of Moderate Intensity and Risk of Type 2 Diabetes. Diabetes Care 2007;30:744752.

4. Henness S. Pharmacologic interventions in the prevention of Type 2 Diabetes. Clin Op in Endocrinology, Diabetes And Obesity
2007;14:166169.
5. Holman RR. Diab Res Clin Pract 1998;40(suppl):S21S25.
6. UKPDS. Diabetes 1995;44:12491258.
7. Nathan DM, et al: Diabetes Prevention Program (DPP). N Engl J Med 2002;346:393403.
8. Chiasson JL, et al: Prevent Non-Insulin-Dependent Diabetes Mellitus (NIDDM). Lancet 2002;359:207277.
9. Buchanan T, et al: Troglitazone in the Prevention of Diabetes (TRIPOD). Diabetes 2002;51(9):279602803.
10. Torgerson JS, et al: Xenical in the Prevention of Diabetes in Obese Subjects (XENDOS). Diabetes Care 2004;27(1):15561.
11. Gerstein H, et al: Diabetes Reduction Assessment with Ramipril & Rosiglitazone Medication (DREAM). Lancet 2006;368:10961105
and NEJM 2006;355:15511562.
12. Nathan, et al: Impaired Fasting Glucose and Impaired Glucose Tolerance (PIOPOD), implications for care. Diabetes Care
2007;30:753759.
13. Harris MI, et al: Is the risk of diabetic retinopathy greater in non-Hispanic blacks and Mexican Americans than in non-Hispanic whites
with type 2 diabetes? A U.S. population study. Diabetes Care 1998;21(8):12305.

54

American Diabetes Association Self-Assessment Program

Educational (Learning) Critiques

Item 62
Answer D
It is widely believed that a meal plan that focuses on the foods the patient is used to eating is most likely to engender
the best compliance. This also fits with the concept that there is no specific prescription for proportion of carbohydrate, fat, and protein combinations that are specifically proven to result in better care of the patient with diabetes.
This also implies that if one tries to change the foods the patient is used to eating, then, by definition they will, most
likely, not be able to follow the diet even in the short-term, or especially the long-term. So nutrition counseling that
is individualized holds the best chance for success in fulfilling the Nutrition recommendations of the ADA.
Extracts from ADA recommendation:
The Dietary Reference Intakes (DRI) report of the Institute of Medicine recommends that, to meet the
bodys daily nutritional needs while minimizing risk for chronic diseases, healthy adults should consume
4565% of total energy from carbohydrate, 2035% from fat, and 1035% from protein. It must be clearly
recognized that regardless of the macronutrient mix, total caloric intake must be appropriate to weight
management goals. Sugar alcohols and non-nutritive sweeteners are safe when consumed within the daily
intake levels established by the FDA The FDA has approved five non-nutritive sweeteners for use in the
U.S. These are acesul fame potassium, aspartame, neotame, saccharin, and sucralose. Before being allowed
on the market, all underwent rigorous scrutiny and were shown to be safe when consumed by the public,
including people with diabetes and women during pregnancy. Substantial evidence from clinical studies
demonstrates that dietary sucrose does not increase glycemia more than isocaloric amounts of starch.
[Patients with] type 2 diabetes should be encouraged to achieve the US Department of Agriculture (USDA)
recommendation for dietary fiber (14 g fiber/1000 kcal) A dietary pattern that includes carbohydrate
from fruits, vegetables, whole grains, legumes, and low-fat milk is encouraged for good health.
Bibliography
1. Institute of Medicine: Dietary Reference. Intakes: energy, carbohydrate, fiber, fat, fatty acids, cholesterol, protein, and amino acids.
National Academies Press, Washington, DC. 2002.
2. Nutrition Recommendations and Interventions for Diabetes. Diabetes Care 2007;30(Suppl 1):S48S65.
3. Franz MJ, et al: Evidence-based nutrition principles and recommendations for the treatment and prevention of diabetes and related
complications. Diabetes Care 2002;25:148198.

Item 63
Answer A
A characteristic syndrome has been described in African-Americans, often seen in patients with new-onset diabetes
who present with DKA. After initial usual aggressive care of their DKA and its precipitant, they are able to be
weaned from insulin and act as typical type 2 patients. It is felt to be a genetically specific atypical diabetes, most
likely related to the ability of the -cell to recover insulin secretion function, possibly initially suppressed by an
undue glycotoxicity or lipotoxicity. African-Americans do not have any particular extra responsiveness to TZDs.
They have an increased incidence of family history of diabetes, as do other patients with type 2 diabetes. There is
no association with sickle cell trait and the expected low proportions of type 2 patients who have anti-islet cell
antibodies. Option (A) is the correct answer, the ability to stop insulin therapy soon after presentation with DKA.
Bibliography
1. Banerji MA, et al: GAD antibody negative NIDDM in adult black subjects with diabetic ketoacidosis and increased frequency of human
leukocyte antigen DR3 and DR4. Flatbush diabetes. Diabetes 1994;43:741745.
2. Pinero-Pilona A, et al: Idiopathic type 1 diabetes. J Diabetes Complications 2001;15:328335.
3. Maldonado M, et al: Ketosis-prone diabetes: dissection of a heterogeneous syndrome using an immunogenetic and -cell functional
classification, prospective analysis, and clinical outcomes. J Clin Endocrinol Metab 2003;88:50905098.
4. Kitabchi AE. Ketosis-prone diabetes: a new subgroup of patients with atypical type 1 and type 2 diabetes? J Clin Endocrinol Metab
2003;88:50875089.
5. Wajchenberg BL. -Cell Failure in Diabetes and Preservation by Clinical Treatment. Endocr Rev 2007;28(2):187218.

Educational (Learning) Critiques

American Diabetes Association Self-Assessment Program

55

Item 64
Answer C
The ADA believes a A1C less than 7% is a reasonable goal for glycemic control. A preprandial capillary plasma
glucose level should be 90130 mg/dL and peak post-prandial capillary plasma glucose l180 mg/dL. The ADA
also recommends that A1C goals should be as low as possible, even l6%, but, most importantly, as low as possible
without undue hypoglycemia. So in this patient option (C) would be the correct answer, a fasting glucose level
l140 mg/dL would not be considered low enough.
The discussion in consensus conferences had to do with the accepted data that damage to tissues starts with
post-prandial glucose levels L140 mg/dL (DECODE, Honolulu Heart Study), and with DCCT, UKPDS, and
EDIC trials we know that decrease in A1C reduces risks of complications, but that risk of hypoglycemia, especially
with recommended therapies of secretogogues and insulin, engender significant risks of hypoglycemia. So at a
societal level, one would not want to cause undue hypoglycemia in a whole population because of goals that might
be too tight.
Table 3. Intensive therapy for diabetes: Reduction in incidence of complications.
Type 1
DCCT
9 R 7%

Type 2
Kumamoto
9 R 7%

Type 2
UKPDS
8 R 7%

Retinopathy

63%

69%

1721%

Nephropathy

54%

70%

2433%

Neuropathy

60%

Cardiovascular disease

41%

16%

A1C

DCCT Research Group. N Engl J Med 1993;329:977986.


Ohkubo Y, et al: Diabetes Res Clin Pract 1995;28:103117.
UKPDS 33: Lancet 1998;352:837853.
Adapted from D. Kendall, International Diabetes Center

56

American Diabetes Association Self-Assessment Program

Educational (Learning) Critiques

DCCT A1C 9.2% vs 7.1% for


Mean 6.5 years; EDICT avg A1C 8.0

Any Cardiovascular Event

0.12
0.10

42% Risk Reduction


P + 0.017

0.08
0.06

Conventional

0.04
Intensive

0.02
0.00

0 1 2 3 4 5 6 7 8 9 10 11 12 13 14 15 16 17 18 19 20 21
Years Since Entry
Number at Risk
Intensive:
705
683
629
113
Conventional: 714
688
618
92

Cumulative Incidence

0.12

Non-Fatal MI, Non-Fatal Stroke or Cardiovascular Death

0.10
0.08
0.06

Conventional

0.04

58% Risk Reduction


P = 0.18

0.02

Intensive

0.00
0 1 2 3 4 5 6 7 8 9 10 11 12 13 14 15 16 17 18 19 20 21
Years Since Entry
Number at Risk
Intensive:
705
686
640
113
Conventional: 721
694
637
96
Figure 7. DDCT/EDIC Trial: Reduction in CV outcomes by glycemic control with insulin therapy.

Bibliography
1. Hanefeld M, et al: Diabetologia 1996;39:15771583.
2. Decode Study Group. Lancet 1999;354:61721.
3. Donahue RP, et al: Diabetes 1987;36:689692.
4. DCCT Research Group. The effect of intensive diabetes treatment on the development and progression of long-term complications
in insulin-dependent diabetes mellitus: Diabetes Control and Complications Trial (DCCT). N Eng J Med 1993;329:97786.
5. Harris MI, et al: Is the risk of diabetic retinopathy greater in non-Hispanic blacks and Mexican Americans than in non-Hispanic whites
with type 2 diabetes? A U.S. population study. Diabetes Care 1998;21(8):12305.
6. American Diabetes Association. Clinical Practice Recommendation. 1999.
7. DCCT Group. Diabetes Care 1995;44:968983.
8. DCCT/EDIC Research Group. The effect of intensive diabetes treatment of cardiovascular disease in type 1 diabetes:
The Diabetes Control and Complications Trial/Epidemiology of Diabetes Interventions and Complications (EDIC) Study.
N Eng J Med 2005;353:26432653.
9. United Kingdom Prospective Diabetes Study (UKPDS) Group. Intensive blood glucose control with sulphonylureas
or insulin compared with conventional treatment and risk of complications in patients with type 2 diabetes (UKPDS 33).
Lancet 1998;352:83753.

Educational (Learning) Critiques

American Diabetes Association Self-Assessment Program

57

Item 65
Answer E
The current ADA guidelines suggest metformin should be started at the onset of therapy for all the reasons
mentioned in this item.
K It improves insulin resistance in liver, decreasing gluconeogenesis and glycogenolysis. It has a high initial
response rate, drops A1C p 2%. There is no initial weight gain and can have a modest initial weight loss.
K It is not associated with hypoglycemia when used alone or with TZD, exenatide, or DPP-4 Inhibitors.
K It has potential to delay or prevent type 2 diabetes mellitus and progression (DPP-2 trial), but its secondary
failure is only modestly better than sulfonylureas (ADOPT trial).
K It decreases MIs 39% in the retrospective analysis in the UKPDS obese subgroup, it decreases advanced
glycosylated end-products (AGEs), and improves endothelial dysfunction.
K Disadvantages One must watch for:
k GI side-effects on initiation, though titration up to maximal effective dose of 2 g/day over a month
decreases the risk.
k Delay initiation until Cr levels are proven stable using radiologic iodinated contrast media studies. This will
reduce risk of lactic acidosis. Moreover, dont use if have significantly impaired hepatic function or cardiovascular compromise (CHF) or Cr L1.4 female, L1.5 male.
k Blood levels of metformin increase in patients with Cr clearance l70 (age L70). Patients with Cr Clearance
l40 is where most cases of lactic acidosis cases have occurred. Thus, clinical judgement must be used in
patients age 7080 (the age the product insert recommends stopping or not using metformin).
D-Glcu so
+e
Protie ( n R )

rodamA i
Procud t

Schif fBa se

HO
HO

HO
CHO
+
+RNH2
NHR
HO
OH
OH
OH
OH
OH
OH

O
NHR
OH
OH

HO

.A G
Cro sslikn de

P
A - -en enoid

HO

[Pro tei ]n
NHR
O

OH

OH

Figure 8. Advanced glycation end products.

NH

NH2

NH

NH2

NH2

NH
NH

CH3
N
CH3

Aminoguanidine

Metformin
(Dimethylbiguanide)

Figure 9. Guanidino compounds that bind dicarbonyls.

58

American Diabetes Association Self-Assessment Program

Educational (Learning) Critiques

O
O
X [Pro tei ]n

OH

Aminoguanidine (AMG)

NH

HN

.E.
Pr to snie

Fructoselysine
Glucose

Serine
L-Threonine
Dehydrogenase

Ene-diol

Aminoacetone
Triose Phosphates
Dihydroxyacetone PO4
Glyceraldehyde-3-PO4
GAPDH
L-Lactate
Pyruvate

Ketone Bodies
Acetoacetate
-Hydroxybutyrate

Methylglyoxal

SSAO

Mono-oxygenase
Acetol
Mono-oxygenase
P = 450
Acetone

Figure 10. Sources of methylglyoxal.

4000

P = 0.03

Media Methylglyoxal
(pmols/gm RBCs)

3500
3000
2500
2000
1500
1000
500
0

+ Metformin
- Metformin
Treatment Group

Figure 11. Media levels of methylglyoxal with or without metformin.

Bibliography
1. Nathan DM, et al: Management of hyperglycemia in type 2 diabetes: A consensus algorithm for the initiation and adjustment of therapy.
A consensus statement from the American Diabetes Association and the European Association for the Study of Diabetes. Diabetes Care
2006;29:19631972.
2. Bailey CJ, et al: Metformin. N Engl J Med 1998;334:574583.
3. DeFronzo R, et al: Metformin Study Group. Efficacy of metformin in patients with non-insulin-dependent diabetes mellitus.
N Engl J Med 1995;333:541.
4. Diabetes Prevention Program Research Group: Reduction in the incidence of type 2 diabetes with lifestyle intervention or metformin.
N Engl J Med 2002;346:393403.
5. Kahn SE, et al: ADOPT trial. N Engl J Med 2006;355:242743.
6. UKPDS Group. Effect of intensive blood-glucose control with metformin on complications in overweight patients with type 2 diabetes
(UKPDS 34). Lancet 1998;352:854865.
7. Rahbar S, et al: Evidence that pioglitazone, metformin and pentoxifylline are inhibitors of glycation. Clin Chem Acta
2000;301(12):6577.
8. Ruggiero-Lopez D, et al: Reaction of metformin with dicarbonyl compounds. Possible implication in the inhibition of advanced
glycation end product formation. Biochem Pharmacol 1999;58(11):176573.
9. Vitale C, et al: Metformin improves endothelial function in patients with metabolic syndrome. J Intern Med 2005;258(3):2506.

Educational (Learning) Critiques

American Diabetes Association Self-Assessment Program

59

Item 66
Answer E
Metformin reduces insulin levels and therefore it would reduce C-peptide levels. In fact, metformin does reduce
hepatic glucose output, hepatic gluconeogenesis, insulin levels, and fasting blood glucose levels.
Bibliography
1. Macfarlane DP, et al: Oral antidiabetic agents as cardiovascular drugs. Diabetes Obes Metab 2007;9(1):2330.
2. Chan. Role of metformin in the initiation of pharmacotherapy for type 2 diabetes: an Asian-Pacific perspective. Diabetes Res Clin Prac
2007;75(3)25566.
3. Bailey CJ, et al: Metformin. N Eng J Med 1996;334:574.
4. Schimmack G, et al: AMP-activated protein kinase: Role in metabolism and therapeutic implications. Diabetes Obes Metab
2006;8(6):591602.
5. Guigas B, et al: 5-Aminoimidazole-4-carboxamide-1-beta-D-ribofuranoside and metformin inhibit hepatic glucose phosphorylation
by an AMP-activated protein kinase-independent effect on glucokinase translocation. Diabetes 2006;55(4):86574.
6. Natali A, et al: Effects of metformin and thiazolidinediones on suppression of hepatic glucose production and stimulation of glucose
uptake in type 2 diabetes: a systematic review. Diabetologia 2006;49(3):43441.

Item 67
Answer B
A general principle of combination therapy for treatment of hyperglycemia in type 2 diabetes without using
insulin is that each agent will reduce A1C an additional 1%. There is a logic, though no published studies, to use
a combination of TZD, metformin and an incretin mimetic to control blood glucose levels avoiding an undue risk
of hypoglycemia. One achieves benefits of glycemic control, potential CV benefit, potential -cell preservation,
as well as decreased endothelial dysfunction and inflammation. Moreover, if exenatide is used, it is likely to achieve
significant weight loss. There is no added benefit in using two TZDs (rosiglitazone and pioglitazone).
Bibliography
1. Davidson MB. Triple therapy: definitions, application, and treating to target. Diabetes Care 2004;27(7):18345.
2. Hamid Z, et al: Triple therapy in type 2 diabetes: insulin glargine or rosiglitazone added to combination therapy of sulfonylurea plus
metformin in insulin-naive patients: response to Rosenstock, et al. Diabetes Care 2006;29(10):2331.
3. Dorkhan M, et al: Glycaemic and nonglycaemic effects of pioglitazone in triple oral therapy of patients with type 2 diabetes.
J Intern Med 2006;260(2):12533.
4. Tran MT, et al: Comparison of the glycemic effects of rosiglitazone and pioglitazone in triple oral therapy in type 2 diabetes.
Diabetes Care 2006;29(6):13956.
5. Rosenstock J, et al: Triple therapy in type 2 diabetes: insulin glargine or rosiglitazone added to combination therapy of sulfonylurea
plus metformin in insulin-naive patients. Diabetes Care 2006;29(3):5549.
6. Roberts VL, et al: Triple therapy with glimepiride in patients with type 2 diabetes mellitus inadequately controlled by metformin
and a thiazolidinedione: results of a 30-week, randomized, double-blind, placebo-controlled, parallel-group study. Clin Ther
2005;27(10):153547.

Item 68
Answer C
Increased quantity of visceral fat is felt to potentiate, by several different mechanisms, muscle and liver insulin
resistance, reduce insulin secretion and is also is directly related to increasing vascular endothelial dysfunction.
In the past, it was felt that insulin resistance was an issue purely of muscle with reduced glucose uptake, and liver with
increased glucose production; but several years ago, it was learned it is about fat, not fat in the buttocks and the
thighs, but visceral obesity, the apple physiognomy, the beer belly, the increased visceral fat that accounts for increased
insulin resistance, and is also associated with diabetes and its complications. When the individual fat cell increases in
size and the total visceral fat quantity increases in volume, they become endocrine organs. They secrete cytokines and
too much of some hormones like angiotensinogen, and too little of others like adiponectin. This affects musclereducing glucose uptake, liver-increasing glucose production, and actually affects the pancreas in reducing further
-cell secretion. Moreover, the body attempts to reduce that volume and quantity of visceral fat by shifting free fatty
acids to the periphery; and the free fatty acids also go to the muscle, reduced glucose uptake, go to the liver increasing

60

American Diabetes Association Self-Assessment Program

Educational (Learning) Critiques

glucose production, also, in fact, reduce -cell function even more and starts the diabetic state. Moreover, the
combination of FFAs, cytokines, and abnormal hormone secretions are directly related to endothelial dysfunction
and inflammatory changes in the arterial epithelium; and this is exacerbated by CRP and PAI-1 production by the
insulin-resistant liver, which all go to the lining of the arterioles and start the atherosclerotic process. Thus, there is
a clear relationship between obesity, insulin resistance, diabetes and atherosclerotic disease.
Bibliography
1. Rader DJ. Effect of insulin resistance, dyslipidemia, and intra-abdominal adiposity on the development of cardiovascular disease
and diabetes mellitus. Am J Med 2007;120(3 Suppl 1).
2. Bergman RN, et al: Abdominal obesity: role in the pathophysiology of metabolic disease and cardiovascular risk.
Am J Med 2007;120(2 Suppl 1):S38.

Item 69
Answer B
Post-prandial hyperglycemia is the earliest detectable glycemic abnormality in the majority of patients with
diabetes; 80% of them present in this way. It contributes to the A1C level of 6.5% to a greater extent than does
fasting blood glucose. It has been associated with an increased risk of microvascular complications in the
DECODE study and Honolulu heart study. It has been associated with an increased risk of macrovascular complications in the NHANES study as well as an interpretation of the DCCT results. Even with A1C at goal, 75% of
post-prandial glucose levels will be elevated and, indeed, with A1C of 6.5% roughly 75% of this value is a result
of post-prandial hyperglycemia. There is one study that suggests that by controlling post-prandial glucose levels
adverse outcomes may be reduced i.e., complications of pregnancy in women with diabetes (decrease large fordates babies by 2/3, decrease C-sections by 2/3, and almost eliminate neo-natal hypoglycemia). However, other
than that study, there is no data that proves that reducing post-prandial hypoglycemia will, in fact, reduce other
complications or cardiovascular outcomes. So, the correct answer is option (B), that no convincing data exists that
treating post-prandial glucose elevations will reduce cardiovascular outcomes or other complications of diabetes.
Bibliography
1. Yamagishi I, et al: Role of post-prandial hyperglycaemia in cardiovascular disease in diabetes. Int J Clin Prac 2007;61(1)8387.
2. Rendell MS, et al: Targeting post-prandial hyperglycemia. Metabolism 2006;55(9):126381.
3. Leiter LA, et al: International Prandial Glucose Regulation (PGR) Study Group. Post-prandial glucose regulation: New data and new
implications. Clin Ther 2005;27(Suppl 2):S4256.
4. Robertson C. Physiologic insulin replacement in type 2 diabetes: optimizing post-prandial glucose control. Diabetes Edu
2006;32(3):42332.
5. Brindisi MC, et al: Post-prandial hyperglycaemia: to treat or not to treat? Diabetes Metab 2006;32(2):105.
6. Gerich JE. Post-prandial hyperglycemia and cardiovascular disease. Endocr Pract 2006;12(Suppl 1):4751.
7. Mecacci F, et al: Maternal metabolic control and perinatal outcome in women with gestational diabetes treated with regular or lispro
insulin: comparison with non-diabetic pregnant women. Eur J Obstet Gynecol Reprod Bio 2003;111:1924.
8. de Veciana M, et al: Post-prandial versus pre-prandial blood glucose monitoring in women with gestational diabetes mellitus requiring
insulin therapy. N Engl J Med 1995;333:12371241.

Item 70
Answer C
Fast analog insulins were developed to mimic normal physiology of prandial insulin, release and duration. Studies
have shown that they can reduce hypoglycemia overnight or during the day. They can decrease post-prandial
hypoglycemia by its rapid onset and loss of effect over several hours. As part of a basal/bolus protocol of insulin
therapy, it allows the patient to eat when they wish, skip or delay meals, eat varying quantity of meals, do shift work,
travel across time zones, wake up different times on weekends and weekdays and in that sense instead of the disease
therapy controlling ones lifestyle, i.e., must eat at the same time everyday, the patient can have freedom of lifestyle,
have control over their disease. Though we would have liked to have seen studies confirming reduction consistently
in A1C, this has not occurred as yet in the published studies.

Educational (Learning) Critiques

American Diabetes Association Self-Assessment Program

61

Bibliography
1. Robertson C. Physiologic insulin replacement in type 2 diabetes: optimizing post-prandial glucose control. Diabetes Edu
2006;32(3):42332.
2. Mudaliar SR, et al: Insulin aspart (B28 asp-insulin): a fast-acting analog of human insulin: absorption kinetics and action profile
compared with regular human insulin in healthy nondiabetic subjects. Diabetes Care1999;22(9):15016.
3. Heller SR, et al: Effect of the fast-acting insulin analog lispro on the risk of nocturnal hypoglycemia during intensified insulin therapy.
U.K. Lispro Study Group. Diabetes Care 1999;22(10):160711.
4. Bruttomesso D, et al: Restoration of early rise in plasma insulin levels improves the glucose tolerance of type 2 diabetic patients.
Diabetes 1999;48(1):99105.
5. Dailey G, et al: Insulin glulisine provides improved glycemic control in patients with type 2 diabetes. Diabetes Care 2004 ;27(10):23638.

Item 71
Answer A
Multiple metabolic mechanisms have now been implicated in the etiology of complications due to hyperglycemia.
Strong data exists in the literature for the role of increased flux in the aldose reductase pathway, production
of advanced glycosylation end products, increased protein kinase C pathway flux, increase flux through the
hexosamine pathway and hyperglycemia increasing oxidative stress in mitochondria. As a result they have multiple
affects in cells and in vascular endothelium and have been strongly associated with the production of eye, kidney,
and nerve disease.
There is also a pentose-phosphate shunt pathway that takes fructose 6-phosphate and creates, via transketolase,
pentose-phosphate. One can give a medicine, benfotiamine, a fat-soluble thiamine relative, that will increase
pentose-phosphate shunt flux and in doing so will reduce flux in several of the above pathways. Thus, the correct
answer is increasing pentose-phosphate shunt flux it can have the potential to reduce complications of diabetes,
not increase them.
q Glucose

NADPH NADP+
NAD+ NADH
q Sorbitol
q Fructose

q Glucose-6-P

Polyol pathway
GFAT

q Fructose-6-P

Gln

Pentose-5TK
phosphates
+
Erythrose-4- Thiamine
phosphate

q Glucosamine-6-P q UDP-GlcNAc
Glu
Hexosamine pathway

NADH NAD+
q DHAP

Protein kinase C pathway

q Glyceraldehyde-3-P
NAD+
Q GAPDH

q -Glycerol-P q DAG

q O2

q Methylglyoxal

q AGEs

AGE pathway

NADH
1, 3-Diphosphoglycerate
Figure 12.

Bibliography
1. Brownlee M. The pathobiology of diabetic complications: A unifying mechanism. Diabetes 2005;54.
2. Brownlee M. Biochemistry and molecular cell biology of diabetic complications. Nature 2001;414:813820.

62

American Diabetes Association Self-Assessment Program

Educational (Learning) Critiques

q PKC

Item 72
Answer D
Improved outcomes in patients with diabetes are seen when the blood pressure is under 130/80 mm Hg.
This was seen in the UK PDS and HOT trials. The recommendation of an ACE-inhibitor (ACE-I) or an ARB is
felt to be important for their additional benefit in reducing microalbuminuria if it exists. Patients with diabetes may
require combination therapy with an ACE-I or an ARB. Although beta-blockers are often needed especially in the
post MI state, it may increase the risk of hypoglycemic unawareness and, in the patient with peripheral vascular
disease, by leaving leaving unopposed alpha sympathetic activity, can result in new symptomatic claudication; thus
the physician must alert the patient in this regard. HCTZ can, in fact, increase blood glucose level but this is felt to
be a relative contraindication and not an absolute contraindication as option (D) states. Thus, the correct answer is
option (D). Thus clinically, in a patient with newly-diagnosed diabetes, one might avoid HCTZ as it may make the
difference between lifestyle therapy and need for monotherapy for glycemic control or between mono and combination therapy for glycemic control. However, in patients already on combination or insulin therapy, its effect on
glycemic control is felt to be minimal and can easily be adjusted.
Bibliography
1. Hansson L, et al: HOT trial. Lancet 1998;351:17551762.
2. UKPDS Group. BMJ 1998;317:703713.
3. Goodman, et al: Textbook of pharmacology [re: sulfonylurea and beta-blocker discussion].
4. Russo D, et al: Am J Kidney Dis 1999;33:851856.
5. Parving HH, et al: N Engl J Med 2001;345:870888.
6. Viberti GC, et al: Circulation 2002;106:672678.
7. Lewis EJ, et al: N Engl J Med 2001;345:851860.
8. Brenner BM, et al: N Engl J Med 2001;345:861869.

Items 7375
Answers 73 (B); 74 (A); 75 (C)
Morbidly obese patients would certainly benefit from weight loss. Weight loss has been shown to decrease blood
glucose levels, decrease insulin resistance, and decrease correlates of insulin resistance including endothelial
dysfunction, inflammatory markers, dysfibrinolysis, insulin levels, blood pressure, and dyslipidemia. According
to 2-year published data (Figure 13), exenatide is the only drug for diabetes that seems to have a persistent effect
on reduction in weight. In 30 weeks of treatment, 25% of patients lost 510 lbs, 25% lost 1020 lbs and 25% lost
L20 lbs. Weight loss seems to continue for at least 2 years.
In general, each additional agent in combination therapy of patients with type 2 diabetes drops the A1C 1% point.
Thus, even if 2 additional agents were added in the patient with a A1C of 9.6%, already on metformin, it would not
bring the A1C to even ADA goals of control. Thus, insulin, as the most effective agent to reduce blood glucose,
is the best option in this example.
A long-distance truck driver with diabetes on insulin therapy can no longer hold a commercial drivers license.
In addition, this truck driver is fearful of injections. Thus, a DPP-4 inhibitor, such as sitagliptin, would be the best
choice with this patient.

Educational (Learning) Critiques

American Diabetes Association Self-Assessment Program

63

Ope
-nal bel
xt snois een
(10 g bid EXE)
82-w eek compel t er (n=31)4
nI t ent-to-treat (n=515 )
0

-1

Change in Body Weight (kg)

Change in Body Weight (kg)

Placebo -controlel d
trials

-2
-3.5

-3

.0 2

kg

-4
-4. 4 .0 3

-5
0

10

20

30

04
50
60
Treatment (weeks )

kg

70

80

-2

Baseline BMI (kg/m 2 )


25 to 30 to 35 to
<25 <30 <35 <40 >40

n=
11
n=91

-4

n=97

n=64

-6
n=
15
-8

-10
90

Figure 13. Change in body weight following 82 weeks of exenatide treatment.


BL = baseline
Blonde L, et al: Diabetes Obes Metab 2006;8.436447.

Bibliography
1. Buse JB, et al: Metabolic effects of two years of exenatide treatment on diabetes, obesity, and hepatic biomarkers in patients with type 2
diabetes: An interim analysis of data from the open-label, uncontrolled extension of three double-blind, placebo-controlled trials.
Clin Ther 2007;29(1):13953.
2. Ratner RE, et al: Long-term effects of exenatide therapy over 82 weeks on glycaemic control and weight in overweight metformin-treated
patients with type 2 diabetes mellitus. Diabetes Obes Metab 2006;8(4)419428.
3. Phelan S, et al: Impact of weight loss on the metabolic syndrome. Int J Obes 2007 Mar 13.
4. Kakafika AI, et al: The role of endocannabinoid system blockade in the treatment of the metabolic syndrome. J Clin Pharmacol
2007;47(5):64252.
5. DeFronzo RA, et al: Diabetes Care 2005;28:10921100.
6. Blonde L, et al: Diabetes Obes Metab 2006;8:436447.
7. Deeks ED, et al: Pioglitazone/metformin.Drugs 2006;66(14):186377; discussion 187880.
8. Wellington K. Rosiglitazone/Metformin. Drugs 2005;65(11):1581-92; discussion 15934.
9. Buse JB, et al: Metabolic effects of two years of exenatide treatment on diabetes, obesity, and hepatic biomarkers in patients with type 2
diabetes: An interim analysis of data from the open-label, uncontrolled extension of three double-blind, placebo-controlled trials.
Clin Ther 2007;29(1):13953.
10. Nauck MA, et al: Sitagliptin Study 024 Group. Efficacy and safety of the dipeptidyl peptidase-4 inhibitor, sitagliptin, compared with
the sulfonylurea, glipizide, in patients with type 2 diabetes inadequately controlled on metformin alone: a randomized, double-blind,
non-inferiority trial. Diabetes Obes Metab 2007;9(2):194205.
11. Blickle JF. Meglitinide analogues: a review of clinical data focused on recent trials. Diabetes Metab 2006;32(2):11320.
12. Riddle MC, et al: Insulin Glargine 4002 Study Investigators. The treat-to-target trial: randomized addition of glargine or human
NPH insulin to oral therapy of type 2 diabetic patients. Diabetes Care 2003;26(11):30806.
13. Raskin P, et al: INITIATE Study Group. Initiating insulin therapy in type 2 Diabetes: a comparison of biphasic and basal insulin
analogs. Diabetes Care 2005 Feb;28(2):2605.

64

American Diabetes Association Self-Assessment Program

Educational (Learning) Critiques

ADA-SAP 1 PDF Version Multiple Choice Questions Section A


1

26

51

76

27

52

77

28

53

78

29

54

79

30

55

80

31

56

81

32
32

57

82

33
33

58

83

34
34

59

84

10

35

60

85

11

36

61

86

12

37

62

87

13

38

63

88

14

39

64

89

15

40

65

90

16

41

66

91

17

42

67

92

18

43

68

93

19

44

69

94

20

45

70

95

21

46

71

96

22

47

72

97

23

48

73

98

24

49

74

99

25

50

75

100
American Diabetes Association Self-Assessment Program

2007 Professional Evaluation, Inc. All rights reserved.

65

Please see page 66 for Program Evaluation Form

ADA-SAP 1 Program Feedback


Thank you for participating in the ADA-SAP Module 1: Basic Principles of Diagnosis and Management of Type 2
Diabetes. If you would like to provide any feedback on this or other Self-Assessment Programs from PEI, please
email your comments to info@proevalinc.com. PEI looks forward to your continued participation in new programs
of continuing education as they become available.

66

2007 Professional Evaluation, Inc.

Anda mungkin juga menyukai